You are on page 1of 82
Ad) aaleilly Ail BL YoY Agta 5 gall — ple Gb a gis gl gS Cita) Faby GL ales AMD aL Yly Mull y ALY gle Aa YL call JG Aedscll obs del 8 cou yall le panadall Sell fb atl Ui pass puatly Glaseyl aby 5 Age g dan! GES (eal) ge pai) as Fae Widens gle Aig nt bib oti) al Leia 1 oes ali gh Lani! eae = Fay) aie Gola gy ALAS paieall we yallede =f EUivall jab soall cigs 9 AMGEN yuiny saaly lal ye SI gee dips 988 Ob i glll Gl sell Me Gal WEY) YI SY ele} ate 955 oh ple Bs yee MAY AG yy arin ie Lal 3 ¢ sll cial =A huyl Say yall JS2 gi Vf Fase WI ys gle GUS pre 8 G3 VB ag Yl yy le Nhe Labia). + EL SD Glas sae 21) MMe VAs ential 0 pdata Mia tse LY Ganealll A OY Gaus Galea sll pe ad sh i ual ¢ sles gre US esti hy glad gs yall asia (sd Ministry of Education and Higher Learning Medicine Colloquium Exam ~- Second Session 2011 IMPORTANT INSTRUCTIONS Each candidate should read this page before answering the questions. The candidate should follow the directions below I 2 3 4- 5 6- 10- He 12- Write clearly your FIRST and FAMILY NAME and APPLICANT NUMBER with ink (pen) in the designated space (colored square). Please use the pencil ONLY. Do not use any pen (like bic) or ink. Completely Fill the answer squares using pencil only. Only ONE answer is allowed to each question Avoid erasing as much as you can. When needed, do erase the cancelled answer COMPLETELY. You may use the question bookle as scratch, and make sure to return it with the answer sheet at the end of the exam. On the answer sheet, you should fill up the square corresponding to the right answer, in pencil only. No writing allowed at all. Keep your answer sheet clean and tidy, do not fold or tear it. The duration of the exam is THREE HOURS. The composition consists of 180 QUESTIONS. Lmportant Note : Any square filled up with a pen or ink cannot be read by the machine. It reads answers filled up in PENCIL only. Thank You & Good Luck Ministére de l’Education et de l'Enseignement Supérieur Examen Colloquium Medecine — Deuxiéme Session 2011 INSTRUCTIONS IMPORTANTES Le Candidat doit lire ces instructions avant de commencer a répondre aux questions et devra observer les directives suivantes 1 2 3 4- 9. 10- dle Ecrire son PRENOM et NOM et son NUMERO D'EXAMEN a Vencre et de facon lisible a Vendroit indiqué (coin en couleur) Utiliser UNIQUEMENT un crayon mine Ne pas utiliser de stylo a encre ou d bille (encre séche). Remplir, sur la fiche de réponse, tout le carré au complet, sans sortir de lignes. Chaque question qui comporte PLUS D'UNE réponse, est annulée. Essayer d'éviter, autant que possible, d'effacer. Dans des cas particulier, il faut TRES BIEN effacer la réponse que vous voulez changer. Pour éviter les erreurs, vous pouvez utiliser la feuille de questions comme brouillon, bien que vous devez la rendre, @ la fin de l'épreuve. Interdiction formelle d'écrire quoique ce soit sur la feuille de réponse en dehors des carrés. Bien conserver la feuille de réponse ; elle ne doit pas étre froissée. La durée de l'examen est de TROIS HEURES. La composition comprend 180 QUESTIONS. Remarque : Chaque carré rempli d’encre ou de toute couleur autre que la mine, ne sera pas compté car la machine qui lit les réponses ne peut capter que le signal envoyé par le carré rempli de MINE. Merci & Bonne Chance ‘A23 year-old University student was admitted with uncontrolled hypertension and numbness in the periphery despite the intake of an ACE inhibitor. Fils primary physical examination was normal except for a BP of 200/120. Laboratory studies revealed serum creatinine of Img/dL, Nat 148 mEq/L, K+ 28 mEq/L. and HCO; 25 mEq/L. Doppler ultrasound of renal vessels was normal. The most possible cause of this, hypertension is: . Renal artery stenosis Cushing Syadrome Primary aldosteronism Pheochrorocytoma Stress from recent exams FoneDp Un etudiant universitaire age de 23 ans a ete admis pour ‘une hypertension non controiee et un engourdissement poripherique malgre la prise d’un inhibiteur de Venzyme de conversion de I'angiotensine. Son examen physique primaire est normal excepte pour une tension arterielle a 200/120. Ies examens de laboratoires revelent une creatinine a Img/dL, Na+ 148 mEq/L, K+ 28 mEq/L and HCO; 25 mEq/L. lultrason avec doppler des vaisseaux du rein sont normaux. La eause la plus possible de cette hypertension est A. Stenose de I'artere renale B. Syndrome de Cushing C. Aldosteronisme primaire D. Pheochromocytome E. Stress a cause des examens recents Gib) Gn gia "Ule aye gaan wi 220M game JL lle ye AM Spe ep 5 yak, La 1 Nae Le asl IS LAN ga Gea ol Baal Ga Ga Vee gal Sakae sabeafad Seen yt S Gf Hy pid Ge TA K+ slfeiSe glle ) ANa+ Sega gl 7° HCOB 5 Ufeise Sabet OF acgaatle Ay s16D Aye SAL ak ba bial gl5Y ee 3S! 9S ha Sia 28 LAB AB iy pagal Aust pyyD BL han pall ye Ais 3. ‘A.40 year old man presents with lateral epicondylitis (ce tennis elbow). Which of the following treatments is likely to be beneficial? Corticosteroids for short -term Corticosteroids for long-term Exercise and immobilization for short-term Nonsteroidal anti-inflammatory drugs for long- term Bracing for short-term Un homme de 40 ans se présente avec une épicondylite latérale (tennis elbow). Quel est parmi les traitements suivants celui qui est le plus probablement bénéfique ? Les corticosteroides & court-term Les corticosteroides a long terme Exercice et immobilisation & court-term des Antiinflammatoire nor-steroidien a long terme Contention court terme Sog> moO e> Ta A Ge gn Ue Oo pe day ea ae gt (eal ge Bio) ole ihe 385 of Soy Ac ra hay pa 8S ch Sabai ye Ole Slab D Ash Sb yanad 6 jal aay E cia ay ‘What is the disease that combines painless oral aphthosis, superficial genital erosions and arthritis? A-Behget's Disease B-Reiter syndrome C-Sarcoidosis D- Lupus erythematosus E-Takayasu's disease Quelte est la maladie de systame qui associe aux signes articulaires une aphtose buccale indolore et des éresions genitales superficielles? ‘A. Maladie de Behget Be Syndrome de Reiter © Sareoidose D- Lupus érythémateux disséminé E- Maladie de Takayasu. oF Up la ean gl Gaal aL f Saale gay gales his Job dye angst oA ey kee WB we C iusles 403 D Als os E “The most frequent cause of anemia is Iron deficiency Vitamin B12 deficiency Folic acid deficiency Hypothyroidism Renal feilure mons> La cause la plus fréquente d’une anémie est A. Anémie ferriprive B Deficit en vitamine B12 C.Déficit en acide folique D. Hypothyroidie E.Insuffisance rénale borderline-high triglyderide levels (150 to 199 mg/cil) is correct? ‘A. They should be screened for metabolic syndrome B. Triglyceride reduction is the primary goal of treatment C. Most should receive fibrate therapy D. Most should receive niacin therapy ‘Quelle est la proposition parmi les suivantes concernant les patients ayant un taux de triglycerides limite entre 150 et 199 mg/dl est correcte ? ‘A. Ils doivent etre depister pour un syndrome métabolique B. La reduction de la triglyceride est le but primaire du traitement C. La plupart doivent recevoir un traitement au fibrate D. La plupart doivent recevoir un traitement a la Which of the following statements about patients with | Ruud dayne 40H CH Lal Ga og! Po eal hel ST) aed je A B12 outs je BB ABs yeas je Goul BL Gai D. GID snail E oe peal 8 Shag pte GA oa + (Que VAAN2+) gt 9) ila Abid SL Be Ge gg ail aA SL a alll sea OLB | on aa ad SA Je pina mally Gd SS ne pete D A prostigmine test 1s done for the diagnosis of Tetanos Botulism Myasthenia gravis, Polymyositis Polyneuropathy moOe> Un test a la prostigmine est fait pour le diagnostic de: A, Tétanos B. Botulisme C. Myasthénie grave D. Polymyosite E._Polynévrite 1a dal Gn Guiting all BSI Ge IDA Ath pad B psd glad Gayl C she gine Gi -D pits uae Deel E ‘4 22 years Young man , suffering from fever and umbilical pain secondarily localized to the right iliae fossa with abdominal defense. What is your initial diagnosis? And what is the treatment? ‘A. Acute appendicitis, antibiotic treatment B. Acute appendicitis and urgent surgery C. Gastroenteritis and antibiotics D, Hleitis and antibioties E, Mesenteric adenitis and Monitoring Jeune homme de 22 ans, douleur fébrile de la FID secondairement localisée aprés une douleur péri ombilicale avec défense de la FID. Quel est votre premier diagnostic? Bt quel est le traitement? A. Appendicite aigue, traitement antibiotique B. Appendicite aigue et chirurgie urgente C. Gastroentérite et antibiotiques D, Hléite et antibiotiques E. Adénite mésenténque et surveillance Tmo Gils Wee GE ay Sa Read gb asia 8 Bl ally 2 be othe ¢ Bo ae Fed pall «pial € al UE IN) yal Acad Gay ha 5a) gdh A, onl Wale dal jay de 5a) al Faye Slstaane yg yee gate Cle Baye lake ll lg Ral yay ine a el moos A 62 year old man complains of perineal discomfort and reports that there are streaks of fecal soiling in his underwear. Four months ago, he had a perirectal | abscess drained surgically. Physical examination shows a perineal opening in the skin lateral to the anus, and a cord-like track can be palpated going from the opening toward the inside of the anal canal. Brownish purulent discharge can be expressed from, the tract. Which of the following is the most likely diagnosis? a- Anal fissure b- Anorectal carcinoma c- Fistula-in-ano d- Pilonidal cyst e Thrombosed hemorrhoids Un homme de 62 ans se plaint de malaise perineale et de souillures fécales sur ses sous-vetements. Quatre mois auparavant, il a eu un abces perirectal draine chirurgicalement. L’examen physique revele une ouverture perineale cutanée dans la partie Jatérale de I’anus et on peut palper un trajet indure comme une corde allant de cette ouverture jusqu'a Vinterieur du canal anal. Des secretions purulentes brunatres s‘ecoulent de ce tractus suite a la compression. Le diagnostic le plus probable est : a- Fissure anale b+ Carcinome ano-rectal c Fistule anale d- Kyste pilonidale e-_Hemorrhoides thrombosees Hypertensive hemorrhage occurs most commonly in: Frontal lobe Cerebellum Basal gengilia Pons Occipital lobe mone> L’hemorragie hypertensive se produit le plus frequemment au niveau Lobe frontal Cervelet Ganglion basal Pons Lobe occipital moag> Fees om Sule Wee dey fe pl Sa Wie dh day Oia | CMs panel Om ela al ed | oe ee Lb a el Lele | gee eS | 8 Se Gall pad gg pales Usa “gh tae Gh all be Jasna eH sea Radice Sq pt lb ye -b 0 47 year old male with chronic low back pain prolonged use of NSAIDs. Secondarily generalized sharp epigastric pain. Diffuse abdormunal defense on clinical examination, First diagnosis: A. Pancreatitis B. Perforated peptic ulcer C. Gastroenteritis, D. Cholangitis, E, Acute cholecystitis, Homme de 47 ans. Lombalgie chronique avec prise prolongée d'AINS. Douleur épigestrique brutale secondairement généralisée. Ventre de bois & l'examen clinique. Premier diagnostic A. Pancréatite B. Uleare gastroduodenal perforé C. Gastroenterite D. Angiocholite E. Cholécystite aigue Se i ee BL Ve pe de) saline Ugh je pains zl geo ld tas aly ee oe EBS gal lai sell pane eee AS apc «phe pia Se he OA ugha Gene eB gate gee sai C ug teal 4) dD Jn ed .E 7 A fixed splitting of S2 is heard in: A. Ventricular septal defect B, Persistent ductus arteriosus C. Atrial septal defect D. Pulmonary stenosis E, Aortic insufficiency Un dédoublement fixe du deuxiéme bruit cardiaque rencontre dans A. Communication inter-ventriculaire B.Canal artériel persistant C.Communication inter-auriculaire D.Sténose valvulaire pulmonaire E.Regurgitation aortique ASE ER Ph eas pal gait OA Las Gy ah BS iN cal dC | sb Gxt D aN) opal OF 2 A Bi-year-old GaP? lady has just delivered a 3.100 grams baby after oxytocin induction and vacuum- assisted delivery. The placenta spontaneously detaches and delivers 6 minutes after delivery of the baby. After the placenta delivers there is a significant blood loss. What is the most likely immediate cause for the postpartum bleeding? Multiparity Retained placenta tissue Instrumented delivery Uterine atony ‘Oxytocin induction mone Une femme agée de 31 ans G3P2 vient d’accoucher d'un enfant pesant 3.100 grams aprés une induction par ocytocine et un arcouchement assiste par le vaccum. Le placenta se detache spontanément et s'accouche 6 minutes apres le sortie du bébé. Apres que le placenta, est sorti il existe une perte sanguine significative. Quelle estla cause la plus fréquente de cette hémorragie immeédiate du postpartum ? ‘A. Multiparité 8. __Rétention du tissu placentaire C.Accouchement avec des instruments D. Atonie utérine E.__ Induction a oxytocin Vib Ga, G3P2 “Ue FV Ge omega Ve Garni any ASE Ay Sy Rapa) Sa A ssc Laally cask oe UB EY Ge GIB Van | Can a Le gly 92 Iai Saas aga Pol ws A SATA | Aepta gees eal BO | SY Gael p 50 gC ol 6D Coe Sn Vb Gin al % You receive at the emergency room a 22 year-old girl for confusion and agitation, On the physical exam, she does not have fever but she has wheezing and bronchial congestion, miosis and diaphoresis. Her TA is 15 / 9, her pulse is regular at 50/ min. You suspect drug poisoning and you want to confirm your suspicion. Which of these tests is the most appropriate? ‘A- Dosage of benzodiazepines in urine B- Blood level of transaminases and salicylate C- Dosage of plasma cholinesterase D- Search for opiates in urine E- Determination of cocaine in blood Vous recever aux urgences une jeune fille de 22 ans, sans antécédents perticuliers, pour des troubles de la conscience avec une agitation A examen, elle n’est pas febrile mais vous trouvez une respiration bruyante par encombrement bronchique, un myosis et une Aypersudation. Sa TA est a 15/9 ; son pouls est régulier 4 50/min. Vous suspectez une intoxication et vous voulez confirmer votre suspicion, Lequel de ces examens vous semble le plus adéquat chez cette fille ? A- Dosage des benzodiazépines dans les urines B- Dosage des transaminases et de la salicylémie CG Dosage de cholinestérase plasmatique D- Recherche d’opiacés dans es urines E- Dosage de la cocaine dans le sang Te Ue Ge a ee Ua gly HS Oe JE ag eg Jah OSB As ans pe ssa Gaal GH y BS Gn gual aly 52) Yad 90+ cgasle yal of Ughinn ue aL S35 GJ Cay 2 peat ag th $ AAD Sh pp EG She pail oe Go 8 dia gi Aes poh BD lanl Se LAE GS Sel aS dee all gf Go) ce ga BS OE SI ce gal mOne> 1% Which of the following tests is the least accurate for detecting Helicobacter pylori infection? A. Urea breath test B. Stool antigen test C. Serology for immunoglobulin G antibodies D. Culture ‘Quel est parmi les tests suivants celui qui est le moins fiable pour detecter une infection a helicobacter pylori ? A. Le test de lurée respiratoire B. Le test de I'antigene des selles, CC. Serologie a la recherche d’anticorps IGG D. Culture a MT pk Ue sal Ge st PAu gl haga ISH Seg a DB Loyd Oe goat aA SB gh all Ce gaa Gani B G4 Sea Se gyal pha Gaal.C G Meal clad diag 9 e25)-D 1 ‘An &-year-old, febrile child is admitied to the pediatric department. Large lymph nodes are palpable on both sides of the neck. The examination reveals pharyngitis and hepatosplenomegaly. No symptoms of anemia or bleeding are observed. Atypical mononuclear cells are seen in the peripheral blood smear. What is the most likely diagnosis? ‘A leukemia B. toxoplasmosis, lymphoma D, infectious mononucleosis E. cytomegalovirus infection Un enfant ag6 de 8 ans qui est febrile est admis dans le departement de pédiatrie des adénopathie larges sont palpables au niveau des deux cdté du cou. L’examen physique révéle une pharyngite et une hepatosplenomegalie. Il n’y a pas de symptémes d/anémie et d’hémorragie. Des cellules mononucléaires atypiques sont trouvées 4 examen d’un frottis sanguin. Quel est le diagnostic le plus probable ? Leucémie ‘Toxoplasmose Lympkome Mononucleose Infectieuse Infection 4 cytomegalovirus mone Gi Oe Gis gus A pe ie Sah cle ya ad DG pa Lam Ce On age SS ABD Gale 38 Haley pal Ags ial oo al 5) po 8 Gal xl ds I Slab WAU 6 GRAY gs ll San DE San Naa FIST Gey ga Ugh al pll| ‘| pale | Lah gu gl B sibel p9-C 693 Shae. J.D VDD paeal Geil GSE 18 Which class of agents can cause dangerous cardiac side effects in overdose? ‘A. Beta-blockers B.Aminoglycosides C.Tricyclic antidepressants D. Thiazide diuretics Calcium channel blockers Quelle classe de médicaments peut causer d’effets cardiaques sérieux en cas de surdosage? ‘A. Béta-bloquants B. Aminoglycosides C.Antidépresseurs tricyeliques D. Diurétiques thiazidiques E.Anticalciques chal AA | S538 sel B Glad OE GI Chae | see .D pm SiS hele LE 7 ‘A 14 month-old boy is brought to the Primary Health Care center because of audible wheezing. The mother's pregnancy and delivery were normal. He has had no previous health problems and his immunizations are up to date. The child looks well, the cectal temperature is, 37.6°C, and there isa clear nasal discharge and intercostal retraction, High pitched wheezes are heard over both lung fields. Which one of the following is the most likely diagnosis? A. Bronchiolitis B. Bronchial asthma . Bacterial pneumonia D, Bronchial foreign body E.Croup Un gargon age de 14 mois est amené au centre de soin primaire pour des wheezing audible. La grossese et Vaccouchement chez. la maman étaient normaux. Cet enfant n’a jamais eu de probleme et son programme d'immunisation est a jour. L'enfant parait bien, la temperature rectal est de 37.6°C, il existe un écoulement nasal, claire et une rétraction intercostale. Des wheezings sont entendus au niveau des deux champs, pulmonaires. Quelles est parmi les suivantes le diagnostic le plus probable ? Bronchiolite Asthme bronchique Pneumonie bacterienne Corps etranger bronchique Croup mon e> GED pe Gh et Vt pe die pal as hg sateen Sia Games A Genel | Sie Go St dca ofS SNS Ua pbb lie Maly Lh Ga yan Sy 6 TVs BN 5 pa 9 cae EY ou IAS le yi Bias Gh gle Sipe Shi udl Gae IS asst gl On gl ot ‘Saas Shaped yA eet eB wetie B) IC gett at ee D Goss E 18 | As-year-old male is brought to your clinic because of | ana lake Glgis Fv pue Gib p=] ¢ar psi. On examination you find round, plastic bend | soy ea al oy ie in the lower third of the ear canal close to the tympanic ad hd 20 G3 hs LL ge membre Youramatectidand wean | sain orc water irrigation and then fast-acting glue on an S58 ie AY il daa es ants Sd applicator Lat Sil pe eho Shy Which one of the following is the best option for Eee ede MA Sid Ses Yd eta removal? MY yk Jal yo ID Seg) oe ‘A. A plastic loop curette through an otoscope $5550 oe B. Referral for removal under anesthesia HS Dy Fy ye OI ARES Sad A C. Grasping with forceps OM) fhe DE we D. Applying acetone to dissolve the object PASE a YAY age dd B Un eniant de 3ans est tre clinique a cause wees In enfant de Bans est amene av i Uy geht ee | d’une douleur au niveau de l'oreille, A examen vous eA He NY sit Sal -D trouver une petite boule de plastique qui est au niveau du tiers inferieur du conduit auditif pres de la membrane tympanique. Vous essayer denlever objet fet vous rYarriver pas malgré plusieurs essais, vous avez utilis en premier lierigation a l'eau, puis vous avez essaye avec de la colle a lenlever. Parmi les suivants quelle est la premiére option pour ’enlever ? ‘A, Une curette en plastique a travers un otoscope B, Le réigrer pour enlever le corps étranger Sous anesthésie €. Enlever avec une pince D. Appliquer de Vacétone pour dissoudre l'objet 78 | Which of following arrhythmias is commonly associated with syncope? Sinus arrhythmia First degree heart block Second degree heart block ‘Third degree heart block Tachycardia Pons> Laquelle des arythmies suivantes est fréquemment associée une syricope? Arythmie sinusale Bloc AV du premier degré Bloc AV du second degré Bloc AV du troisi¢me degré Tachycardie roOe> @ pe Gee GIS EAD EAD GG Hues te ASI Ga gl pL Ag yal Ge gall BOM kal Go ll one Seuss moAe> D Which of the following is the preferred diagnostic test in a woman with a low mean corpuscular volume and suspected iron deficiency anemia A. Serum iron B. Serum ferritin C. Transferrin ferritin saturation D. Serum transferrin receptor Quelle est parm les suivantes le test diagnostique preferable chez une femme qui a un volume corpusculaire moyen bas et une suspicion d’anémie ferriprive ? A. Fer sérique B. Ferritine serique Saturation de la ferritine transférine D._Recepteur du transferine sérique aia oS Gani ID On gt iB Slay Uae Shay S ane Wl lay Sa Ses plas A eet oss 8B 298 VAIS Gal yy ELA -C ead Gy jah ities -D a Contraindications to lithium administration include (one answer) ‘A. an administration in combination with chlorpromazine (Thorazine) B. the presence of a renal disease C. any occurrence of the symptoms of schizophrenia D. the presence of depression E. an administration in combination with imipramine (Tofranil) ‘Les contraindications a l'administration du Lithium sont {une seuie réponse): A. Une administration en combinaison avec le Chlorpromazine (Thorazine) La presence de maladie rénale Symptomes de schizophrenic La presence de depression Une administration en combinaison avec Imipramine (Tofranil) moa Talay (als cle otagS go SA athe A (e313) ghey B Aaah ale sya gC ayerD nie ct iia ga 8S Really athe E (sass) 22 ‘A 32 year old woman comes to the office complaining of palpitations. On examination, she has fixed split S2 and a mid flow murmur at the upper left sternal border. ECG, reveals a right axis deviation. What is the most likely underlying cardiac disorder? Mitral valve prolapse Moderate-to-severe anemia Ventricular septal defect Atrial septal defect Puimonic stenosis mONs> Une femme de 32 ans se présents pour palpitations. A examen physique, on note un dédoublement fixe de B2 et un souffle systolique au niveau du rebord sternal gauche au 2eme espace intercostal. Quelle est la pathologie cardiaque la plus probable? A. Prolapsus de la valve mitrale B, Anémie modéréea sévere Communication inter-ventriculaire D. Communication interauriculaire E, Sténose pulmonaire Sei SUE FY Bw aa] ASH SZ ald Unt Uysal yIS seal SI Ken HH 8 alates | SS ih ad Gu PS Sata ga Legal gy yee Gall © th hat Cae gi Aid pla SS A | whee # wht ala Be Waa jal ae pay Seat 2 ‘Compression of the neural elements facing the L2 vertebra can result in all of the following except ‘A. Entrapment of L2 root B, Entrapment of the Cauda equine C. Entrapment of the LS root D. Cord compression Une compression neurologique face a L2 peut coceasionner tout saut ‘A. Une atteinte de la racine L2 B. Une atteinte de la queue de cheval C. Une atteinte deta racine LS D. Une atteinte de la moelle [Higa a ad pag | f hae le IS oH gaye SF Se L2 L2 ue cli) A, ew OS Cua B L5 ot gas C pu koad DD Bell 28 ‘You are consulted in your clinic by @ 40 year-old man for mononeuritis. He doen’t have cardiovascular risk factors. You note in the history of the patient: Many episodes of febrile oligoarthuitis for 5 years, ‘An episode of orchi epididymitis one year earlier, He was operated, 2 months ago for cholecystitis, which ‘was acalculous. What is the vasculitis that best fits this clinical picture? ‘A- Behcet's Disease 1B. Churg-Strauss © Wegener's Disease D- Polyarteritis nodosa E- Microscopic polyangiitis, 'Vous étes consulté dans votre clinique par un homme de 140 ans, sans facteurs de risque cardiovasculaire, pour une ‘multingvrite. A interrogatoire, vous notez: quill présente, depuis 5 ans, des épisodes doligoarthrites fébriles quill acu, un an auparavant, une orchiépidiidymite quill a été opéré, il y a2 mois, pour une chokécystite, qui slest révélée étre alithiasique, (Quelle est la vascularite qui correspond fe mieux a ce tableau clinique ? A> Maladie de Bebget B- Chorg et Strauss Maladie de Wegener D- Périartérite noueuse E-_Polyangéite microscopique. une iyi Ge soy "Ue to ope dey Hphe y ialb ge Jaye a Got galel eam pl Sy oe On AUD atid hl oo Spi Rasa Ba Bee roy Gems hall Gs = wee Oe! Be hale Wee al = pre ote ie 8a ge lly GP de EIT gh Le ee raeen) cing eh A ws sistst B pay els C pile sake iho he D goie gles et -E a ‘A 26-year-old female presents with a T-year history of recurring abdominal pain associated with intermittent diarrhea, 5-7 days per month. Her pain improves with defecation. There has been no blood in her stool and no ‘weight loss. Laboratory findings are normal, including a CBC, chemistry profile, TSH level, and antibodies for celiac disease. Which one of the following would be ‘most appropriate at this point? A. Colonoscopy B. An upper Gl series with small-bowel follow- through A gluter-free diet Educate her about her condition Ask her to drink fluids myo Une ferme agée de 26 ans se présente pour une histoire de douleur abdominal recurrente depuis lan. Cette douleur est associée a une diarrhée intermittente 5 a7 jours par mois. Sa douleur s'améhiore avec la defecation, li n'y a pas de sang dans les selles et pas de perte de poids, Les examens du laboratoire sont normaux, incluant une numeration formule, un profil chimique, taux de TSH et les anticorps pour la maladie coeliaque A ce point quel est parmi les suivants celui le plus approprié ? ‘A. Colonoscopie B. Unrepas baryté avec un transit du grele C. Une diete sans gluten D. Education concernant son état E._Lui demander de boire des liquides wale gt oe AS "Uke Th yee Tt N~O ght lead Q+ il je Lie le 3 3S Se 50 Se bed cee el ol 53 DAE Ge GS Vy Haak gh po ad Ga anal yd) Sn ya STSEL gs snes loa olad wl Sg SN oo gl gil eS aah, 1 Rap a GS alt GS sith A pete eed 8 lel ya! ALL B BaBSh GD Aadis Qe gg sl OSD Oe JS ee LC ells ce its. Boe a8 lee ll E 2 Whick of the following drugs is recommended asa first- line therapy for patients with hypertension and diabetes? Lisinopril (ACE) Atenolol (beta-blocker) Verapamil (Isoptin) Amlodipine (Amlor) goe> Quel est parmi les medicaments suivants celui qui est recommiande comme une ligne de traitement pour les patients ayant un diabete et une hypertension ? A. Lisinopril (ACE) B, Atenolol (beta-blocker) C. Verapamil (Isoptin) D._ Amlodipine (Amlot) Wile bs p LOA Gg hog eee EE) Oe Cg Gl ate pl Sp Be (ACE) dese A (Ge cela) Solna B (28358) Set 8.C stl) oe 6D B13 Ey Conditions associated with an increased risk of colon cancer include all of the following except Ulcerative colitis Diverticulosis Crohn's disease of the colon Gardner's syndrome Villous adenoma of the cecum mons> Les conditions associeess a un risque augmente du cancer du colon comprennent tout ce qui suit SAUF A. Colite ulcereuse. B. Diverticulose C. Maladie de Crohn colique D. Syndrome de Gardner (polypose farsiliale) E._Adenome villeux du cecum, SR) GB ad gon | We GGL SS Gy)8 th Cie ce as DA, eS! B oad i ay 8 lac peje & poe oe pes gt sE 28 | Which one of the following is the best option for a ged 8 Lal pw Al gl supporting a sprained ankle? sda A, Tape Grell bye B. Cast : C. Semi-rigid or lace-up support yy lege has ek D. Blastic bandage ta hy D Quelle est parmi les suivantes la meilleure option pour / Ja contention d’une entorse de la cheville ? i ‘A Ruban adhésif B Platre C. Unsupport semi rigide D. Bandage elastique _ _ 29” | A two year old girl was well until 12 hours ago, when | de 1¥ Jd sae Gals Slike ae lb she begins to develop lethargy, vomiting, and episodes of intermittent cries during which it seems to have abdominal pain, During the examination, it develops a bloddy mucus stools with jelly aspects and a painful abdomen. The most likely diagnosis is: A-pyloric stenosis, B-Appendicitis, C-Urinary tract infection DeIntussusception E-Peptic Ulcer Une fillette de deux ans allait bien jusqu' il y a 12 heures, quand elle commence 3 développer une \sthargie, des vomissements, et des épisodes de cris intermittents durant lesquels elle semble avoir des douleurs abdominales. Durant I'examen clinique, elle développe une selle couleur marron et a un abdomen douloureux. Le diagnostic le plus probable est Ac Sténose du pylore B- Appendicite C- Infection urinaire D- Invagination intestinale E- Uleére peptique AS yi AB lt Oe lad clay Lise DIS pl al oye gaalle le Aadaie sy pola Gate ale By IS sal A tae JST ee J glee whe Ges A 8) ys B Bp gee al ge iD dma E 30] A 63 years ole Man, presenting with occasional rectal [a2 chy gw St) le Fe pe dap bleeding since three months and diarrhea alternating — | Jk jie Shas! gx Jhpely jel? le wit constipation wih weightloss of 10X55 Fst JAN pesky ye &S agnosis: Us he A ‘A. Colon cancer ye B. Infectious colitis Pe etd OB C.Hemorthoids eS D. Anal fistula est ont D E. Hyperthyroidism Sebi bs E Homme de 63 ans, depuis 3 mois rectorragies ‘occasionnelles et alternance diarrhée constipation avec amaigrissement involontaire de 10 kgs. Premier diagnostic ‘A. Cancer dus colon 8B, Colite infecticuse . Hemorroides D. Fistale anale E Hyperthyroidie 37_| Which one of the following statements about older (pod ful ae GD al gh patients with cholecystitis is correct? ‘A. Murphy’s sign is an accurate indicator of cho.ecystitis B, Most have back or flank pain C. Allhave nausea 1D, More than one half of patients are afebrile ‘Quelle est parm les propositions suivantes, la Proposition exacte, concernant les malades ages ayant, ‘ane cholecystite ? ‘A. Le signe de Murphy est un indicateur fiable de la cholecystite B. La plupart ont des douleurs de dos ou du flane C. Tous on des nausées D. Plus quela moitié des patients sont afebrile TB hed Oe Sally nll gb AS Mae BS poy GA tye We OLA Sash hele AS Batt oll Oe Cle pecbee B eal ORE Go oles ppt PN EY om yl Geet Ge ISD Beis @ Which of the following statements about herniated disk is correct? ‘A. Twenty percent of patients with acute low back pain havea herniated disk B. Few patients with herniation have a sciatica CC Herniated disc is seen only in patients with symptoms of sciatica D. Physical examination findings are not useful in localizing the level of the disc herniation Quelle est la proposition exacte concernant une hemie Biscale ? A. Vingt pour cent des patients ayant une douleur Jombaire aigue basse ont une hernie discale B. Quelques patients ayant une hernie ont une sciaticalgie C._La hemie discale est trouvée seulement chez des patients ayant des symptomes de sciatique D. Les données de 'examen physique ne sont pas utiles pour localiser le niveau de la hernie discale wo ah ae I ed eg] Sis SA GAD gla Oe Healy yy ote rhe eb did Soa Soe | Sie ee | He SE aed ON ge palo» LBB Woe | Geto Se Bi Fit Go gd LC ail Gye Gal je! pg Cll spel bade et Le ped Glee yal SD | 33 | Which one of the following therapies is most likely to FO Bey GE GIS Oe Gl improve outcomes in patients with ankle sprains? et els Ge Gish Gal an al A. Therapeutic ultrasonography + ais B. Application of heat ee gual kA C. Non-steroidal anti-inflammatory Jjod Gals. B D. Immobilization boys iS sae oe ly aC Quelle est parmi les therapies suivantes celle qui sl. améliore le plus souvent les patients ayant une entorse dela chevitle? A. Ultrason thérapeutique B. Application de chaleur C. Antiinflammatoire non steroidien D._Immobilisation 34° | A 23 year old student complains of sore throat for three | ZG Ua day Ge 84 "Lhe YT o jac Gul | days. He started with a runny nose and a slight cough. ‘The symptoms are a little better this morning, Which of the following is most likely? Rhinovieus Neisseria gonorrheae Hemophilus influenza Corynebacterium diphteriae Streptococcus pyogenes (A) moos Un jeune étudiant de 23 ans se plaint de douleur a la gorge depuis3 jours, Ses symptomes ont commence avec un écoulement nasal et une légere toux, Il va un peu mieux ce matin. Quel est la cause la plus probable ? Rhinovirus Neisseria gonorcheae Hemophilus influenza Corynebacterium diphteriae Streptococcus pyogenes (A) monger pale alk Dany Gil Dns Lay ld AGW UAT ye gh cal ois hy aid Se JS AM ost A 6 & = mone 3% ‘A39-year-old female presents with lower abdominal/pelvic pain, On examination, with the patient in 2 supine position, you palpate the tender area of her abdomen, When you have her raise both legs off the table while you palpate the abdomen, her pain intensifies. Which one of the following is the most likely diagnosis? A. Appendicitis B.A hematoma within the abdominal wall musculature C. Diverticulitis D. Pelvic inflammatory disease E, Anovarian cyst Une femme agée de 39 ans se présente pour des douleurs abdominales pelviennes basses. A 'examen, la pationte étant en decubitus dorsal, vous palpez une zone dovloureuse au niveau de l'abdomen. Quand vous lui demandex de soulever les deux jambes de la table de Vexamen alors que vous lui palpez son ventre, sa douleur augmente, Quelle est parmi les suivantes le diagnostic le plus probable ? A. Appendicite B. Un hematome au niveau de ia musculature de la paroi abdominale ©. Diverticulite D. Maladie inflammatoire pelvienne E._Kyste ovarien Ge GA Asie TO pe a Spm) shal se ghd GE, aad Gi) pit foal eli evay G3 sail | Ue ho ey atl Ge Ld Haba lel Sd cad Gam A DLA le Ses SN am a tayo 5) ig A Dla oe Gale gr es Bo Agha Sal he C alg ede D | vee pS E | ‘What type of breast cancer is the most common and to shat location? ‘A- Ductal, superior external quadrant B- Cystosarcoma, inferior internal quadrant C- Lobular, superior internal quadrant D- Medullary, superior external quadrant E+ Cystosarcoma, inferior external quadrant Quel type de cancer du sein est le plus fréquent et dans quelle localisation ? A. Canalaire, quadrant supero-externe B. Cystosarcome, quadrant infero-interne C. Lobulaire, quadrant supero-interne D. Médullaire, quadrant supero-externe E,_Cystosarcome, quadrant infero-externe ee IM go ghee ee Tears! etl gall og sh A AIS Sh oI SES OB RI gall OM aepamad -C wat gall gD cod lid GI daa Se E B17 ea Which of the following is more consistent with the diagnosis of epidydimitis than with testicular torsion? ‘A. Fever, urgency and other urinary symptoms B. Recurrent episodes of pain . Abnormal cremasteric reflex D. Exacerbation af pain with elevation of the testis (Prehn sign) ‘Quelle est parmi Jes suivantes celle qui va avec le diagnostic e’epidydimite plutdt qu’ une torsion testiculaire ? A. Fiovre, symptomes urinaires et symptomes durgence urinaixes, B._ Episodes de douteur récurrentes, C._ Reflexe cremasterien anormal D._ Exacerbation de la douleur avec 'élevation du testicule (signe de Prehn) pas Ee Ul BT IS On gh Spas gall eo tie ad ie | eA iy cals ey aA 8 oj Sie Sb AB | Balad pals pt baie (or Se) Seek gy oe A ED 38 A 65 year-old man develops superficial thrombophlebitis in multiple sites including the arms and chest, He has had several episodes in the past couple of months, each of which lasted few days, Which of the following neoplasms is most closely associated with this patient's clinical problem? Prostatic carcinoma Lung carcinoma Pancreatic carcinoma Acute promyelocytic leukemia Paroxysmal nocturnal hemoglobinuria mon s> Un homme age de 65 ans develope des thrombophlebites superficielles en plusieurs endroits comprenant les bras et le thorax. I! a eu plusieurs. episodes similaires dans les derniers mois. Chacun a persisté plusieurs jours. Quel est parmi les neoplasmes suivants qui le plus probablement est associe au probleme clinique de ce patient ? Carcinome prostatique Carcinome pulmonaire Carcinome pancreatique Leucemie aigue promyelocytique Hemoglobinurie paroxystique nocturne moos yb a Le Gel ae Saat Ql gh alee pais opine daa SUS ally Sayed Lage aly SS scam Oped SS a RID SI Ge gh ed tame © Raped yan pl USES ny tne A BY tha B AE EEC ala Gib yi Gu Gola). AA he sagt LOE 7! Bos 35 Which one of the following findings is an indication for imaging in patients with acute low back pain? A. Failure to respond to conservative therapy after hwo to three weeks. Bilateral leg weakness. Sciatica in one leg. Positive result on straight leg reise ons Quelle est parm les suivantes celle qui est une indication d’imagerie chez les patients ayant une douleur lombaire basse ? A. Echee de reponse au traitement conservateur apres 2a 3 semaines B. Faiblesse bilaterale des deux jambes CC. Une sciatique au niveau d'une jambe D._Resultat positif en soulvant la jambe ea Se ppl yee sph did go ts ol oe Glee Gold EDU gue gal Sa SLT LOA | lind gal ow Gia SA Caled oS aa B 0 Which of the following statements about mammography is correct? A. Digital mammography is diagnostically more accurate in all women B, Digital mammography is more sensitive in women with denser breasts C. Mammography should be discontinued at 70 years of age D, Mammography is as effective at reducing mortality in younger women as in older women Quelle est la proposition exacte concernant la mammographie ? A. Mammographie digitale est plus précise dans le diagnostic chez toutes les femmes B. Mammographie digitale est plus sensitive chez les femmes ayant des seins denses C._ La mammographie doit étre arréter aprés age de 70 ans D. La mamunographic est aussi efficace a réduire la ‘mortalité chez les femmes jeunes autant que chez, les femmes agées, ee oa UO yl © getel gah PSI get 52D ayes DA ele JS 2 FSV pass. BS A SD eed GAD nye IB GES gat i Lad we Gul, LED GD ype Soe Gt Gay C “Ue Ve we ae BBS get gall pat gD | LCL we Shag a yanict eal B19 co ‘A 23 year old female patient comes to the ER with intense and continuous 10/10 pain in the right iliac fossa accompanied by severe nausea. She mentions a1 week delay of menstruation with a positive pregnancy test What is the diagnosis to suspect first? A. Appendicitis B, Ectopic pregnancy ©. Renal cotic D. Irritable Colon E, Colon cancer Une ferume de 23 ans se présente aux urgences pour une douleur de la fosse iliaque droite d’emblée tres intense 10/10et continue accompagnée de nausées importantes. Elle notait un retard de régles de 1 semaine avec un test de grossesse qui est revenu positif. Quel est le diagnostic a évoquer en premier ? A. Appendicite B. Grossesse Extra Utérine C. Colique néphretique D. Colon irritable E._ Cancer colique agp Gyan Le Was Lape 8D on $5 phy os ghd Gi Ba Nf ney ae al al Head ays Te cet fat ABE ge pled das aad ge tals fe gal Cus Yi stad est go Le ej clgl oA pice de B GI uate ee oss -D ois tee OE @ Tha patient with cerebral tamor and intracranial hyperension, papilledema is ‘A. unilateral, on the side of the brain tamor B. unilateral, on the contralateral side of the brain tamor C. pulsatile D. bilateral E. macular Chez un patient ayant une tumeur cérébrale et hyperension intracranienne, I'edéme papillaire est: A. Unilatéral du méme c6té que la tumeur B, Unilatéral, du coté opposé a la tumeur C Patsatile D. Bilatéral E.Maculaire EM) GEG Oy Ge Gln ae oe Cad! Dye Sy Gch As hi A sp nedl El pyle ald SL A pad Sed AL wg all B gua aks cola fae moa (% Which one of the following is NOT an absolute contraindication to vaginal delivery? A, Complete placenta previa B, Primary herpes simplex virus with active genital lesions C. Previous classic uterine incision D. Treated human immunodeficiency virus infection Quelle est parmi les suivantes celle qui n’est pas une contraindication absolue a l'accouchement par voie vaginale ? ‘A. Placenta previa absolue B. Virus d’herpes simplex primaire avec lesion genitale active C. Antecedent d'incision utérine classique D._Une infection de HIV traitée Ge Lp a gl Pig 23 Aaghall LIS CLA, eb Oe eh Sy SelB ALOU ALU ilo | She Ses any SEC go Aes De eye Hah lan 42] Tn temporal arteritis, select from the following signs the oe geual Gye yl Oa wrong answer. 2 URS yell gl A. Headache laa A, B Sudden blindness coh ae B C. Polymyositis with elevated CPK 5) we oa D. Claudication of the jaw CPK gi) gs aaa ghie Gigill < E Fever ae te Dans lartérite temporale, choisir parmi les signes suivants la réponse fausse. A. Céphalées | B. Amaurose fugace C- Polymyosite avec CPK éleves D- Claudication deta machoire E: Fidvre 45 | Chorionic villus sampling for genetic diagnosis is Ds Ge tue Ub je Gk Uned performed between: 13-15 weeks of gestation 16-18 weeks of gestation 10-13 weeks of gestation 14-20 weeks of gestation soap La biopsie de villosite chorionique pour le diagnostic, genetique est fait entre A. 13-15 semaines de grossesse B. 16-18 semainas de grossesse 10-13 semaines de grossesse D._14-20 semaines de grossesse Fa tly ved asd Ge E gael ONT Saad Ge Egeel VL Sad Ge Enel HY Sad Ge al VED | % ‘A 63 year old women comes to see you with a proximal muscle weakness and general bone pains, Her calcium. and phosphate levels are slightly low and her alkaline phosphatase level is raised. Which one of the following 1s the single best possible diagnosis? A. Hyperparathyroidism B Myeloma C.Osteomalacia D. Osteoporosis E. Rickets Une ferme agée de 63 ans se présente avec une faiblesse proximale des muscles et une douleur généralisée ‘osseuse. Son calcium et son phosphore sont légerement diminiués et le taux de phosphatase alkaline est éleve Quelle est parmi les suivantes le meilleur diagnostic possitle? ‘A. Hyperparathyroidie B, Myelome C. Dsteomalacie D. Osteoporose E_Rachitisme ene yes Ue TB we Td Sapa Cf hans Lake Ny phen Sg) U8 Kain Stil pt AEH a I ge gl dae Sth T Scinall aga gh GaN gle LLG b BA eros B Bad og.C ke 42L.D chs E a For which of the following findings, a referral to an ophthalmologist is recommended? A. Ciliary flush B, Preauricular lymph node enlargement C. Conjunetival hyperemia D. Purulent discharge Pour lequel parmi les suivants, une consultation chez un ophtalmologiste est recommandée ? ‘A. Des flaches ciliaires B. Ganglions augmentes de volume en periauriculaire C. Hyperémie conjonctivale D, Ecoulement purulent_ Ste Reape Gay SI TT Ge © yall ust yal A Ba Ji fall Gi pa. B Lat 5 Une femme agée de 27 ans nulligravid présente une dysménorrhée sévere et une douleur occasionnelle durant les relations sexuelles. Elle pose 50kg et elle a une taille de 160em. L’examen pelvien est normal Léchographie révéle un ovaire gauche de 2x3em et un ovaire droit de 4x6em, Quelle est parmi les suivantes la cause la plus probable de cette symptomatologie ? Appendicite chronique Endométriose Syndrome de congestion pelvienne Maladie des ovaires polykystiques Syndrome prémenstruel noNS> Waihe JY Gay Se WV la yee yal] et dyn ak pee Ge ols seed Lebgey 450+ Mgijy aaa Sra Sn oad pth aa Gass | GN inal y pT pall sagll Bel oll Gad a ID Ge gh pst $ ial de Bj ASA (eo gi SB ad Mad LC SRD anne gatas ela D ted JBL E | a Besides the pancreas, amylase is produced in the following organs except A. Salivary glands B. Liver ©. Kidney D. Fallopian tubes E. Brain Outre le pancréas, I'amylase est produite par les organes suivants saul: ‘A. Glandes salivaires B Foie Rein D. Trompes de Fatlope E.Cerveau SS oe DAN gil nl Gat Gale ll ae Le 8 eLiac Aa od A as) B 4s C So cout D gua E 50 ‘A 67 year old man who has poorly ~controlled type 2 diabetes for 10 days presents with a superficial, painful, purulent ulcer on the sole of his foot associated with minor trauma from the footwear. He has no other acute signs or symptoms of illness. Which of the following pathogens is the most likely cause of the infection? ‘A. Peptococeus magnus B, Staphylococcus aureus . Escherichia coli D. Clostridium perfringes Un homme age de 67 ans un diabete de type 2 qui n’est pas contréle pendant 10 jours se presente avec un wleere plantaire superiiciel purulent et douloureux qui a ete associe a un trauinta minime de sa chaussure. II na pas & auties signes ou symptOmes aigus de sa maladie. Quel est parmi les suivants le germe le plus probablement responsable de cette infection ? A. Peptococcus magnus B. Staphylococcus aureus C. Eschetichia coli D. Clostridium perfringes GE ed On gly Ue Wee day or Sia ell 5 pte a ll ye at The tba le Byes al asc ye Sa Se his ae Sy catel J sole gyal «12h 4a Gal ll DS ASD Ra jad! Lb pl ge gt oye SOS Na dal age weet a a SSA, Sua Lall Sp pba SS. 5g SN LS, 2yI.C ‘abla D a ‘4S year-old woman consults you for fatigue and ‘weight loss. You discover in her past medical history that she had early menopause at the age of 35 years, soon after giving birth to her last child. Her current symptoms include nausea, early post-prandial fullness, and diarrhea. She does not smoke. Her only medication is Calcium with Vitamin D (500 mg/400 IU). Among the following, which would be a priority to rule out? ‘Adrenal insufficiency Hypothyroidism Celiac disease ‘Adenocarcinoma of the pancreas Hypercalcemia mone> Une femme agee de 45 ans vous consulte pour une fatigue et perte de poids. Vous decouvrez dans son histoire medicale passee qu’elle a eu une menopause precoce al'age de 35 ans, & ceci aprés avoir accouche son demier enfant. Ses symtpomes actuels ieluent une nausee, une sensation de plenitude precoce post prandiale et diarrhee. Elle ne fume pas. Le seul medicament qu’elle est entrain de prendre est le calcium avec Vitamine 1 (600 mg/400 IU). Parmi les diagnostics differentiels suivants, quel est celui qui doit étre exclue le premier ? Insoffisance surrenalienne Hypothyroidie Moladie celiaque Adenocarcinorne du pancreas Hypercalcemie mone> Aaa 5 URI Call "Le fo Le ae pe RSS BG Apts Lp ot Saw ae Ubi gad) LD Aa idle JAY WAV ay Vy3h Mle To Gee Sie Di age Gaus GH peel gL Biba ge hy leds hie gl SG oe D Gu ee pss 0 unl Lael vest Ge sl (A 29 as tone tall ASI yak Bp BLS Gai B gle lac SE gd tl poe ge pay Dems b BE fee ‘A 76 year-old man complains to his physician that for six | ‘months he has developed regularly severe abdominal pain one to two hours after eating, So bad is the pain that he is Josing weight because he is afraid to eat. There is no blood in his stool, The most likely diagnosis for his, problem is: Diverticulosis Regional enteritis, Imitable bowel syndrome Mesenteric angina Amyloidosis, mone> Un homme age de 76 ans se plaint chez son medicine que depuis 6 mois il develope segulierement une douleur abdominale sévére et ceci une a deux heures aprés le repas. La douleur est tellement importante qu'il est entrain de perdre du poids car il a peur de manger. Il na pas de sang dans les selles, Le diagnostic le plus probable de ce probleme est A. Diverticulose B. Enterite regionale C. Syndrome du colon irritable D. Angine mesenterique E__Amyloidose We Se ab aud Sis le Vie ye day pte ot ake oe SE ay oat Doped JH ae ele I Reb ig Se us Ey TS eye Il Id eae TSE on a Sal Lee JU yeh tal cade A geal oly lel B. ead asia LC den Sis -D cA ll la SE 33 _| What are the most conimon causative germs in the PID Ted a Mp (Pelvic inflammatory Disease)? Pad het ‘A. Chlamydia and Gonococeus Sub Sa LA BLE coli and Proteus, Lied, Liat chs nen —B | C Klebsiella and Pseudomonas cropyear ic D. Staphylococcus aureus and Peptostreptococeus = tau ioied cleat D E.Pseudomonas and Chlamydia SORE A 8 SIN Quele germes sont le plus fréquernment en question Sid ap E dans les PID (Pelvic Inflammatory Disease)? ‘A. Chlamydia et Gonocoque BLE. coli et Proteus, Klebsiella et Pseudomonas D. Staphylococcus aureus ot Peptostreptococcus E. Pseudomonas et Chlamydia o Which one of the following test is recommended for all, patients with syncope? A. Echocardiography B. Blectracardiography C. 24hour holter Monitoring D. Head -up-tilt-table testing Lequel des tests suivants est recommande pour tous les patients ayant syncope ? Echocardiographie Electrocardiographie Monitoring par holter durant 24 heures Un test de surelevation de la table , la tete haute Dop> (papal sie calls GIS Sl alll Ge gl © pt os Orbe GAD lll pave dudes. A PaaS Sb ele VE saab pligl Maal ye ABI yall Sd gle Esa GA iad Gant cS Hypercalcemia is associated with: A. Non-Hodgkin's lymphoma B_ Squamous cell lung cancer C. Hodgkin's lymphoma D. Squamous cell head and neck cancer E, Hypernephroma L'hypercalcémie est le plus souvent associée @ ‘A. Lymphome non-Hodgkinien B. Cancer epidermoide du poumon C.Maladie de Hodgkin D. Cancer epidermoide de tete et cou E, Hypernéphrome eels Be Lal Seo vA al A Ub B Lesied USeae C FD! gh D aay ok gS E wa SA we 36 ‘A.55 year-old man is evaluated for weakness, Over the past few months, he has noted slowly progressive weakness and cramping of the left leg. Lately, he has also had some trouble swallowing food. He is awake and alert, Findings on neurologic examination are normal except for marked atrophy with fasciculations in the muscles of both leg, hyperactive reflexes in the upper and lower extremities, a diminished gag reflex, and 4 positive extensor plantar response. Which of the following represents the most likely diagnosis? A. Cervical spondylosis, B. Guillain-Barré syndrome C._Lambert-Faton syndrome D. Vitamin B12 deficiency E, Amyotrophic lateral sclerosis, Un homme age de $5 ans est evalue pour une faiblesse. Durant les quelques demniers mois, ila note une faibless2 progressive et des crampes au niveau de sa jambe gauche. Dernierement il a eu des troubles a la deglutition des aliments. Il est alerte et vigile, Liexamen, neurologique est normal excepte pour une atrophie ‘marquee avec fasciculation des muscles des deux jambes, des reflexes hyperactifs au niveau des extremites superieures et inferieures, une diminution du reflex nauséeux, et une reponse plantaire positive en extension, Quel est parmi les suivants celui qui represente le diagnostic le plus probable ? Spondylose cervicale Syndrome de Guillain Barre Syndrome de Lambert-Eaton Deficit en vitamine B12 Sclerose amyotrophique late moOS> Bia Os Gin Ue 98 nye Oey yon yb Bagh duel gS) Gans CaS sm daa Beis CAD iiaae gh pala gly jue “Lind Aad June Ligtels N36 Le asin cpt Gea Asia soil Diane G8 ge Gilet! yale Auge GLAM Gk ae bus oS Bilal «peed Sei GLE silky el Ga GI Anas dba i Ayla Sage 8 ASV) a ae 448) ah eI. phe Dux LDUAB Os — Cee Le iC B12 cats 55¢-D payne ula aE 7 ‘A 40 years oldman presents for ineapacity of abduction of the shoulder following a direct fall. The x-rays are normal. He probably has a ‘A. C6 paralysis B. C7 paralysis, C. C8 paralysis D. Tear of the rotator cuff Un patient de 40 ans se présente pour incapacité de Vabduction de l’épaule suite a un trauma direct. Les radios sont normales, II est probable qu'il souttie de ‘A. D'une paralysie de C6 B. D'une paralysie de C7 C. D'une paralysie de C8 D._Diune rupture de la coiffe des rotateurs io pe SEU Nee day epee ob plea ae $5 ey MHS an clo es BUS of Lain oe gaia po latt “ C6 as CT gale C8 ae Soywl USI GS ones Eo From which of the following laboratory data is it possible to differentiate between secondary and primary hypothyroidism? A. alow serum Ti level and normal TSH levels B.an clevated serum Té level and low TSH levels Ca low serum Té level and elevated TSH levels D. an elevated serum TA level and elevated TSH levels Parmi lesquelles des données de laboratoire suivantes il est possible de différencier entre une hypothyroidie secondaire et une hypothyroidie primaire ? ‘A. un taux bas de T4 sérique et un taux normal de TSH sérique B.un taux élevé de Ta sérique et un taux bas de TSH sérique C.untaus bas de sérique et un aux elevé de TSH serique D. un taux élevé de T4 sérique et un taux élevé de TSH serique Taped Slagle Ge gh of MEY Bp BLS alt Ge Grill Sy at Fl Sp Be cay 5 5 Suey GaSe glans T4 yy sine A, eel TSH sy Qin Ghe T4 gs Sue B aise TSH is Fiasey nie hina TA 5 sinsa CO we, TSH essing gly Glhne TA shee D ee» TSH 38 Which one of the following recommendations about antibiotics for the prevention of cold symptoms is correct? A. Antibiotics are recomumended for patients with purulent nasal discharge after three to five days, B. Antibiotics are recommended to prevent pneumonia C. Antibiotics are not recommended for patients with a suspected cold. D. Antibiotics are recommended for adults with cold and fever after three to five days. Quelle est parmi les propositions suivantes celle qui est exacte concernant Ie traitement des toux relies a une gripe ? ‘A. Les antibiotiques sont recommandes pour les, patients qui ont un ecoulement nasal de 3a 5 jours B. Les antibiotiques sont recommandes pour preventt une pneumonie C. Les antibiotiques ne sont pas recommandes pour Jes patients ayant une gripe suspecte D. Les antibiotiques sont recommandes pour les adultes qui ont une grippe et fievre depuis 3a 5 jours Zab Lee GCL Cal pl Cal Lastly) Shale spss ADE se ye lad ee A Be gh ok Go gan Ge (LD ed, eo 8 Sayeed ya Had eB u coe al Uy hla ie .C po S33 gd ON WG ue LSM Ayo ad ae sD EN Ly ASI peilee) Ge pul Reed sal 60 ‘patient presenting with headache and a visual perception of halos around lights can be secondary 10 which of the following conditions? A. Meningitis 8. Temporal arteritis, C. Acute angle glaucoma D. Venous sinus thrombosis Un patient se présente pour des céphalées et une perception visuelle de Halos a la lumiere, ceci pout étre secondaire a laquelle des conditions suivantes ? A. Meningite B. Artérite temporale C. Glaucome a angle aigu D__Thrombose du sinus veineux B28 SH 939 ET Oe Oe SSE AG Galel us GI Se te pal bm $A NAD Yo gle gts ae ys. A exel cbt Sia B Be 915 GGL S29 2 ED 67 60 year old man. With Inguino-scroial mass suddenly SF MSI Sen Le Ws ope Day became hard, painful and irreducible with arrest of eb pute lg sg ad ad a stools and gases. What is your initial diagnosis? Zl ga scl jialy Jad a A. Epididymo-orchitis +N B, Strangulated inguinal hernia 4 3 Comes sews eos aaa A D.Syphilitic chancre Geel SB E, Testicular Torsion ad eed leah OC cobb as D Homme de 60 ans. Masse inguino-serotale ancienne wake devenue soudainement irréductible et douloureuse avec arrét des matidres et des gaz. Quel est votre premier diagnostic? A. Orehi-épidlidymite B. Hernie inguinale étranglée C. Orchite virale 1D. Chanere syphilitique E. Torsion testiculaire w ‘A2 year Old child is febrile to 39 degrees Celsius. A urine analysis shows: 3RBC/mm3 and culture grows 100,000 colonies of E. Coli You would: ‘A- admit the child for acute pyelonephritis B-Start oral antibioties for upper urinary tract infection C-Do a PPD test to eliminate a urinary tuberculosis. D-Repeat the urine culture and await the outcome, ‘E-Repeat the urinalysis and start intravenous antibiotics Un enfant de 2 ans est febrile a 39 degrés celsius. Un examen cytobactériologique des urines (ECBU) fait par poche montre : 3GB/mun3 et une culture qui pousse en "100000 Colonies aE Coli Votre attitude serait ‘A- Admetire enfant pour pyélonéphrite aigué B- Commencer une antibiothérapie orale pour infection urinaire haute CC Faire une [DR a la tuberculine pour éliminer une tuberculose urinaire De Refaire YECBU et attendre le résultat. B+ Refaire ' CBU et commencer une antibiothérapie intraveincuse. ATR Sm CB) 8 Oke «pe ile fal HG pl saad Ge Oa Gb pans Ves + 3CB/mm3 FAS pla Sor Sag OLS, Yl SED es git I ib Jat A Be vans US gded JAY tye Ghbaw call B “shal Ayah Sau Gylull sla Jato Jeti elyal -C isd al els Sql KL Syd g p5 4221 -D ogome sletaay oly Sil Gilas sate! -E eas B29 Co A 15-year-old boy with a persistent asthma that is poorly controlled on a short-acting inhaled beta2 agonist presents for routine evaluation. Which one of the following is the most appropriate next course of action? ‘A, Add a leukotriene inhibitor B, Add an inhaled corticosteroid C. Adda second ~generation anti-histamine D. Add a long-acting beta2 agonist and leuktriene inhibitor tun gargon age de 1Sans avec un asthme persistant qui est mal controlé par les agonistes beta 2 par inhalation se presente pour une evaluation de routine, Quelle est parmi les suivantes le plan d’action le plus approprié ? A. Ajouter un inkibitevs de leukotriene B, Ajouter un corticosteroide en inhalation C._ Ajouter un antihistaminique de 2eme generation D. Ajouter un agoniste beta 2.a longue durce d'action et un inhibiteur de la leukotriene Ys Gee a Oe gl Tle 18 pe phat Yap Sin LD Ge ans eis asd dal ge pian Alla rod Rae SY) IGN Daath ge lil Ge Gt Sale LGA el a6 ge S558 BLL B Tek Sis Guntigs shine Hla. fe la SIS Yip ke 48 DD OSS! co Which one of the following recommendations about antibiotics for the treatment of URI is correct? ‘A. Antibiotics are recommended for patients with, purulent nasal discharge after three to five days B. Antibiotics are recomimended to prevent pneumonia C. Antibiotics are not recommended for patients with suspected URI D. Antibiotics are recommended for adults with URI ard fever after three to five days Concernant les antibiotiques pour le traitement d’une infection des voies respiratoires hautes, quelle est parmi Jes recommandations suivantes celle qui est correcte ? A. les antibiotiques sont recommandes pour les patients ayant un ecoulement nasal purulent apres 3.a 5 jours B._lesantibiotiques sont recommandes pour prévenir tune pneumionie Ces antibiotiques ne sont pas recommandés pour les patients suspects d'une infection respiratoire haute D._ les antibiotiques sont recommandés pour les adultes avec une infection respiratoire haute ot une fidvre aprés 3 a 5 jours Kiyo) Chala Ge AVS Sal Gu gl esd Geel a os Had DD (oe ll Lyall Chabal placid een ae ood il ot Ole Gal pl Rad ODS ed hiya) Iabanall asad eas Se se Uyad islanal hits quale Gab GY Syn si Oe Galle pease fgg eas GUSH iyypad haal ph Qu D Sage Aas Ge US Oe sda Gal MGI ED ay sha, w 35 years old with motor vehicule accident and contusion on the left side of the abdomen, presents to the E.R with pallor cold extremities , hypotension at 8 and tachycardia at 130. What is the organ mast likely injured? A. Spleen B Stomach C.Small intestine. D. Rectum, E. Left colon Jeune de 25 ans accident de la voie publique avec contusion du coté gauche de abdomen, arrive aux urgences avec paleur, froideur des extrémités, hypotension a 8 et tachycardie a 130. Quel est l'organe le plus probablement 1638? A Rate B, Estomac C Intestin gréte D. Rectum! E. Colon gauche os Gale a eas Le 18 ope Day a hall Co Sel deal AS Bay gad Ge ls hy es ah AS Bg fly A deduce pals8i} St J) the FSU La ped a 6 Which of the following statements regarding hypertrophic pylorie stenosis is valid? A. the disease is manifested during the first few days of life B, metabolic acidosis is characteristic C. the disease is manifested during the first 4-6 weeks of life D. bilious vomit is a characteristic finding Quelle est parmi les suivantes la proposition exact, concernant la stenose hypertrophique du pylore ? ‘A. Lamaladie se manifeste Durant les quelques premiers jours de la vie B.L'acidose métabolique est caractéristique C.La maladie se manifeste durant les 4 8 6 premieres semaines de la vie D. Des vomissements bilieux sont earacteristiques Geel ay eee GR ILD Ge gt F pale Cd ce AMT AM DS Gadd GSA Bed elie nan Mall a B bd A= t I) DR Ge yah att C eal shal ya Ge 9h gslateal el yl. 7 A 21-year-old nulligravid woman who is not using contraception has had irregular menstrual periods since menarche at age 13 years. She has noted increased hair growth on her face and lower abdomen. On pelvic examination, there is copious cervical mucus and. slightly enlarged irregular ovaries, Which of the following is the most likely cause of these findings? ‘Adrenal adenoma Idiopathic hirsutism Ovarian tumor Pituitary adenoma Polycystic ovarian disease mone> Une femme agée de 21 ans nulligravide qui n’ utilise pas des contraceptifs a cu des périodes menstructies irréguliéres depuis sa menarche a I'age de 13 ans. Elle a note une augmentation de s9 pilosite au niveau de sa face et de la partie inferieure de abdomen. A V'examen pelvien, il existe un mucus cervical, copieux et des ovaires augmentés de volume d’une facon irrégulitre. Quelle est parmi les suivantes la cause la plus frequente de ce tableau? A. Adénome surrénalien B. Hirsutisme idiopathique Tumeur ovarienne D. _Adgnome de hypophyse E,__ Maladie des ovaires polykystiques Sis Ut) guy pie Taye dys ly Oe gla wan le pac Y ety Dan chee et] SA ye gb AS Ska LIT ee yest oe oat oll Udy Yeas le col Se Bie Jobless eel PASS pA nl Byes SAB eel ipa baa 5 99s. ChaaGll sate aba ola E 8 530 year old man operated at the age of 15 years of peritonitis, with arrest in elimination of stools and gases, with generalized abdorainal pain, cramping and biliary vomiting . What is your initial diagnosis? A, Gastroenteritis B, Occlusion of the colon, C. Crohn's disease Dadhesions intestinal Occlusion E, Bowel cancer Homme de 50 ans opéré a l'age de 15 ans de peritonite appendiculaire, vient pour arrét des matigres et des Baz avec douleur abdominale généralisée a type de crampes et vornissement fécaloides. Quel est votre premiet diagnostic? ‘A. Gastroentérite B. Occlusion du colon C. Maladie de Crohn, D. Ccclusion intestinale par adhesion E. Cancer du gréte Uw tale Ocul le ©? wpe Uns tsa Re dal ga ite 10 oye US Des 539 Ss ee c AL ae gee tonal odd hye E B32 69 | A'56 years old female with intense epigastric stabbing cei al Ge pli bale OM La ae slat pain. Fever and hypotension. Temperature 383039 | Sl yeas TAT Yi Jya a SS degrees. First diagnosis: PANT gags 1 omy ‘A. Gastritis y B. Acute cholecystitis B C. Acute pancreatitis : D. Gastroesophageal reflux disease Ae LS ell E. Hepatitis seta gona gS YF ele. es la OE Femme de 58 ans. Douleur épigastrique intense en coup, de poignard. Fievre 4 383 et hypotension 3 9, Premier diagnostic A. Gastrite B. Cholécystite aigue ©. Paneréatite aigue D. Reflux gastro-cesophagien E. Hépatite 70 | Vitamin D toxicity may produce Tod D Guts han gaye ot US A. Bone pain A B. Duwhea an C. Hypercalcemia Rarerers D. Hypophosphatemia Baye D E. Increase intracranial pressure cat libel ploy La toxicité de la vitamine D peut praduire: ‘A. Douleurs osseuses B, Diarrhée C. Hypercalcémie D. Hypophosphatémie E,_ Augmentation de la pression intracranienne 71 | Which of the following laboratory tests is recommended HS YI La all Ge gt for children classified as overweight who have no risk factors? A. A fasting lipid profile B. Aspartate transaminase and alanine transaminase C. Fasting blood sugar D. Creatinine Quel est parm les test de laboratoire suivants celui qui ‘est recommande pout des enfants classifies comme ayant un exces de poids et qui rant pas de facteur de risque ? A. Profil lipidique a jeun B. SGOT et SGPT C. Clycemie a jeun D. Creatinine Dose aap Lay BSH g5 38 GaN a BP le pat Gah Stal oa Jad 9 OB | iF eB oanC ons SD 72 Which of the following is the initial imaging study recommended for the evaluation of patients with left lower quadrant abdominal pain? A. CT without contrast media B. CT with oral contrast media C. CT with oral and intravenous contrast media D. Ultrasonography Quelle est parmi les imageries suivantes celle qui est recommandee initialement pour évaluer les patients qui ont une douleur abdominale au quadrant inferieur gauche ? A. Scanner sans produit de contrast B. Scanner avec produit de contrast oral C. Scanner avec produit de contrast intraveineux et oral D._Ultrasonographie AU Aged a a BOB Oe gl Se call eB pat Sal oe thal $F hd Ge el SiN gt ils day 32 Gagne hth pee A 9 Si dey GE gyn pits Gane B 123 Lal daney ob gage git pee ory ana} ahs D 7 The presence of red blood cell casts in the urinary sediment is suggestive of Acute hemorrhagic cystitis, Glomerulonephritis Adenovirus cystitis, Acute prostatitis Pyclonephritis, moUAs> résence de eylindres hématiques dans le sédiment urinaire suggére A. Cyslite hemorthagique B. Glomérulonéphrite C. Cystite a Adenovirus D. Prostatite aigue E,_Pyélonéphite eat ey gt Lae Se 8 ay Ce Sia OLS GU A Sg leat B Be pts teas eC Bhs Sines 9» ga -D 0) Kaya el E 7 The test of choice for immediate evaluation of an acutely swollen scrotum is ‘A.a pelvic radiograph B. radionuclide imaging © color Doppler ultrasonography D.CT sean E.MRI Le test de choix pour évaluer rapidement un gonflement serotal aigu est Radiographie pelvienne Une scintigraphie Ultrason avec Dopple: couleur CT scan IRM Ipone> Ghee iil jl pal aka Gena Py wee gels ALA Gal als ppt B cosh gah gina bss C ost sie oD geotlite Oy pai -E 75 Which is the most likely toxicity associated with excessive doses of theophylline? A. Bradycardia B. Drowsiness then coma C. Hepatotoxicity D. Paradoxical bronchospasm E Seizures Quelle est la toxicité la plus probablement associée a des doses excessives de theophylline ? A. Bradycardie B, Somnolence puis coma C. Hépatotoxicisé D. Bronchospasme paradoxal E. Convulsions eo Wad bes IN ee © olla eit pte oe wa gh OA tye bday B Ras tee eal tes D analogs E 76 [in the management of hypercalcemia, the following | Ly talus Se Sa cay idl On gl measure should be avoided oa eas ‘A, Intravenous Furosemide gy amy A B. Thiazide diuretics haa C. Saline infusions eats” D. Intravenous bisphosphonates Sate pf Une E, Intravenous calcitonin as iste ston D F423 a seISE Dans fa conduite 4 tenir devant une hypercalcemie, quelle est 1a mesure que vous devriez eviter ? ‘A. Furosemide en intraveinen B. Diuretiques thiazidiques C. Infusions de serum sale D. Biphosphonates en intraveineux E_ Calcitonine en intraveineux 77 Which one of the following initial findings is most aseful for excluding a diagnosis of giant cell arteritis? ‘A. Normal sedimentation rate B. Normal complete blood count C. Normal examination of the temporal artery D. Normal eye examination Quelle est parmi les suivantes celle qui est le plus utile pour exclure un diagnostic d’zxtérite a cellule geante ? A. Vitesse de sedimentation normale B. Numeration formule normale C. Examen normal de ’artere temporal D. Examen normal des yeux Taal go Saw LD all Ge gh SU he ob tt ets pb oe UA cert GAS 42 ibe 6 a B cee eta Cbs yas gash Ge ask ‘Antibiotics used in the neonatal period should have the following characteristics except one, which one? ‘A-Be bacteriostatic, B-Pass strongly through the dura C-Do not be toxic to the newborn; D-To give intravenously; E-be the cheapest possible Les Antibiotiques utilisés en période néonatale doivent, avoir les caractéristiques suivantes sauf une, laquelle? A. Etre bactériostatiques; B- Pouvoir traverser les meninges; C- Ne pas étre toxiques pour le Rouveau-né; D-Se donner par Voie intraveineuse: E: Etre le moins chers possible. ae Gye IED ge Oy hia al Quen GS J gna bo lat slip oS Bae ye bk BY) Gana ee SY oh Bay oho fib jab Gc Laka OG mones 7 ‘A b2-year-old feinale presents with acute breathlessness of one hour duration. She smokes occasionally. On examination, she is sweaty and needs to sit up, has oxygen saturations of 89% on air, a temperature of 37.5°C and hasa respiratory rate of 30/min ‘Auscultation of the chest reveals widespread expiratory wheezes and extensive bilateral basal crackles. Which ONE of the following is the SINGLE MOST likely diagnosis? ‘A. Asthma| 8, Preumonia C. Pulmonary embolism D, Pulmonary fibrosis E. Pulmonary edema Une femme agée de 62 ans se présente pour un essoufflement aigu et gui dure depuis une heure. Elle fume occasionnellement. A I'examen, elle transpire et a besoin de se mettre debout, elle a une saturation doxygone a 89% a Tair libre, une temperature de 375°C et a une fréquence respiratoire de 30/min. L’ausculation du thorax révele des wheezings expiratoires espacés et des crépitants extensifs basals bilateratix. Quelle est parmi les suivantes le seul diagnostic le plus probable ? A. Asthme B.Pneumonie C Embolie pulmonaire D, Fibrose pulmonaire E.Oedeme pulmonaire UAB ope ie iat le 4 la pe Al chp] sie isle He ga Ree Se le | AS I Gl Clas fe bd Gat Geil Wn 0 OWAS SaaS glad) aay 26 fF ih Janey PAV YG os IIS ple paid fg peal gal GBD ge gh apd Cola ARE le flu bBo Gat ol 6) 3B ag Bo paral C goo bt.D pads E hysteroscopy ? ‘A. Gas embolism B. Hypernatremia C. Uterine perforation D. Fluid overload Des propositions suivantes quelle est celle qui n’est pas une complication de Uhysteroscopie A. Embolie gazeuse B. Hypematrémie C. Perforation utérine BO] A 28 year-old patient is seeking medical advice because | gb Gua Gly Lie VA spac Gaye of incidental finding of a 3.cm right adrenal mass by CT) uaa ya gt ill GS 58 pa Ge scan that was done for a benign Cl reason few weeks | 35g jy) Gill Gal we PG wy ago. The patient is asymptomatic, does not have history | “aS. Sh ce gail Sal egal of malignancy, is not hypertensive, does not have 29° 77) By Cushing features. What will you do next for this patient? | 722 004! Os alo) Sar gael ‘A. Reassure to al Bees Ga Base ye baie B. Follow-up with CT scan after 6 months et aS alae taal Gay chan CUB | CC. Pheochromacytoma should be ruled out ad BS ne SLT D. Surgical resection © E_ ENA of the tumor sliiisel A se parm agi cose ane Andis! B Un homme age de 28 ans cherche un avis medical a ely cause d'une decouverte fortuite d’une masse PAB ayy telos CS surrenalienne de Sem sur an CTscan qui a ete pratique dant DO pour un probleme gastrointestinal begnin et ceci depuis Mugs iyaty E quelques semaines. Le patient est asymptomatique, il n’a nee ° aucune histoire de malignite, il n’est pas hypertendu, et iln’a pas de signe de cushing. Quest-ce que vous faites ‘comme prochaine etape chez ce patient ? \ ‘A. Rassurer le pationt B, Suivi du patient avec un CTscan apres 6 mois C. Un pheochromocytome doit etre exclus D. Resection chirurgicale E, Ponction biopsie de la tumeur ‘81 | EKG signs of hyperkalemia include alll of the following, Ail ygS LS LDS Gay except: S 1ae Le ob Le US pall pale Gilby A. Diminished or absent P waves P clay GEA B. Peaked T waves ie Tota B C. Prolonged P-R interval : + © Promise wane PR antiga Une E. Widened QRS complex ore ae Les signes electrocardiographiques d’une hyperkaliemie incluent tous ce qui suit SAUE A. Diminution ou absence des ondes P B. Des ondes T pointues C. Interval P-R prolongé D. Des ondes U proéminentes E__Elargissement du complexe QRS. 82 | Which one of the following is NOT a complication of SEE Gl & ol» GN os gt eo ia feat “A Spwinebs -B ges sil C GAT Just -D D._Hyperhydratation [83 | perinatal infection (all of the following are true except luge yes one) Noe U4 524 she 4 ima mate pretietion: - G8 EA ie A B-accompanied, in most cases, with fever sya eect vole ‘Can raise the serum C-reactive protein; ee mee é D-is sometimes preceded vy a matemal infection ne ~ Ganscmenme Petes . Css E-Not a specific symptomatologie Mealy Waren G38 ola Gees gt -D WH se Liinfection périnatale (toutes les propositions suivantes er alel Ge ot a OE sont justes sauf une): A> Avune prédilection au sexe masculin; B- S‘accompagne, le plus souvent, de fives; C- Peut élever la Protéine C-réactive sérique; D- Est parfois précédée ’un etat infectieux maternel; E-_N’a pas une symptomatologie spécifique. #4 [Acute aortic dissection should be suspected in a patient) ciara se a yg! abo Gy gis a with: Fe pla A. Anterior chest pain wet gue STA B. Interscapular pain Tks etl B C. Diastolic hypertersion with an aorticinsurficieney | cca ae play murmur Oi D. Allof the above na E. None of the above tment oD fare yf el Gaal IS -E Une dissection aigue de ¥ aorte est suspectée en cas de: ‘A. Douleur thoracique antérieure B. Douleur interscapulaire C. Hypertension diastolique avec un souffle d’insuffisance aortique D, Toutes sont justes E._Aucune n'est juste 8S] Bacterial vaginosis is best treated with: to Anse Sle ley De Land of ‘A. Mebendazole Jpslakaa A B. Tetracycline AB ©. Metronidazole Usslany sec D. Cefuroxime E, Penicillin oe ° Une vaginite bacterienne est le mieux traitee avec A. Mebendazole B. Tetracycline C. Metronidazole D. Cefuroxime 2 Tenn 4 8 ‘\ patient has to undergo splenectomy for hereditary spherocytosis. Which of the following immunizations is indicated prior to that? ‘A, Mumps vaccination B. Pneumococcal vaccination C. Measles vaccination D. BCG Bacillus Calmette-Guerin) E. Tetanus toxoid Un jeune homme doit subir une splénectomie pour sphérocytose héréditaire. Laquelle des vaccinations suivantes est indiquée ? A. Oreillons B. Pneumococcique C. Rougeote D. BCG (Bacillus Calmette-Guerin) E, Ystanos ak Jail Glee el) aie sal B23 845) ol pot Se 5 IS Ba SS Abang gy AA Aa Ye gl Gis gt A apace B Leas! cla C (os = Suk ene) Jat ct .D 9 cE a All the following viruses may be responsible for mononucleosis except A. Epstein Barr virus B. Adenovirus C. Human immunodeficiency virus D. Cytomegalovirus E. Rubella virus Lequel des virus suivants n'est pas associé avec une mononucléose? A. Virus Epstein Sarr B. Adenovirus C. Virus de l'immunodéficience humaine D. Cytomegalovirus E_ Virus de la rabéole Bye a8 OI OS RE a Ss lie Le sib Sly OD oe Je OBE ona A B c Wb Uys -D E Eo ‘A pulsatile abdominal mass is discovered during a routine examination of a 62 year-old woman with hypertension. Ultrasonography reveals an aortic aneurysm diameter of 3.5em, Which one of the following actions is the best next step? A. Yearly ultrasonography B. Referral to a vascular surgeon CC. Immediate surgical repair of the abdominal mass D. No further testing is required Une masse abdominale pulsatile est decouverte durant tun examen de routine chez une femme agée de 62 ans ayant une hypertension artérielle, L’échographie revele un anévrisme aortique de 3.5em de diametre. Quelle est Ja meilleure action parmi les suivantes a faire ‘A. Echographie annuelle B. Referer la patiente chez un chirurgien vasculaire CC. Reparation chirurgicale immédiate de cette masse abdominale D._Aucun test supplementaire est nécessaire anil JUS Gaga Gaby AES opp S| FB Ue TY Uae ey gists po eh she als yl hk gli) | sham I On gt gate ae Sel | © ail pate pine bbs A Balsa Gh gesleniSl Cth Saal 5B est! aha aS 48 Gal pp eC 5 sl le sad cl yal Gye 92.D. 89 Tout ce qui suit est cohérent avec le diagnostic de migraine sauf ‘A- Caphalées fluctuantes et intermittentes B- Cephalées sévéres et continues C- Cephalées battantes D- Histoire positive de céphalées en farnilie E- Bonne réponse a ’aspirine All of the following are consistent with the diagnosis of migraine except ‘A- Fluctuating and intermittent headache B- Severe, continuous headache C- Pounding headache D- Positive family history of headache E- Good response to Aspirin Ua ee dale gh We 8 he ciltes chain Ele A el ing Yb Eline B (Gin a) fa Ela C fia! ble yD ed 80 | Perianal pain with defecation and sitting down Bijan 308 Sec oth laws ol associated to fever. First diagnosis: 1 OPN peel au A, Anal abscess Det B. Hemorrhoids are 4 Anal fistula ate D. Perianal condyloma ay E. Anal fissure co de yids -D wet E Douteur péri-anale constante a la défécation et a fa position assise avec fidvre. Premier diagnostic A. Abees anal B. Hémoroides .Fistule anale D. Condylome péri-anai E. Fissure anale 87 [All the following urolithiases are radiopaque except TO ge ah a AI A aad Cystine stones Calcium oxalate stones Struvite stones Urete stones, Calciuin phosphate stones monaD Les calculs urinaires suivants sont radioopaques saul: Calculs de eystine Calculs d’oxalate de calcium Calculs de struvite Calculs d urate Calculs de phosphate de calcium => oO he te Cine Chee A pom CLS) Cees B SB Sheen C 1s Sheen -D pape hind pas OE 2 ‘A 32-year-old woman is diagnosed to have Trichomonas. She is entirely asymptomatic. Appropriate management should be to ‘A. reassure patient that ne treatment is necessary B. treat regardless of symptoms CC. repeat cultures after the next menstrual cycle and treat if the organism is present D. treat only if the patient is pregnant E. treat only if symptoms are present Une femme agée de 32 ans est diagnostiquée comme ayant du Trichomonas. Eile est completement asyaptomatique. La conduite a tenir appropriée doit atre ‘A. Rassurer la patiente qu’aucun traitement n’est nécessaire B, Traiter sans tenir compte des symptomes ©. Répéter Ia culture apres le cycle menstruel prochain et traiter si Vorgarisme est présent D. ‘Traiter seulement si la patiente est enceinte E,_Traiter seulement si les symptomes sont présents (Gel Gah Sle TY a ye all | cet J) ual gel Ge gl Ya jis Pell tk Soy day Yah dey yall at ale ge hil Gat Sohal B A yg Sal a gj tel C ny Gf Heyl AaRee AS 18 Ae i is Saad. te ty yt wal je! Gas Il bib 4g Baall E a in the case of hypospadias, what intervention is absolutely contraindicated: A; perform VCUG (vesico -urethral cystography) B- Make a circumcision CMake an abdominal ultrasound D-Do a urine culture E-Operate before 18 months Devant un hypospadias, quel est le geste absalument a ne pas faire Faire une cystographie rétrograde Faire une circoncision Faire une échographie abdominale Faire une uroculture Proposer l'opération avant 18 mois moO > Pe as Ge gl gD DD ay 1 "Ullne aed aetla he pet GS ol aKB Ga Spall 38 ota! =C dx E25 taat-D Ve VAS tule ol IE 4 ‘Which one of the following statements about treatment of nonalcoholic fatty liver disease is correct? ‘A. Metformin (Glucophage) reduces hepatic transaminase levels and improves ultrasound findings B. Metformin reduces the likelihood of progtession to cirrhosis C. Metformin reduces cardiovascular mortality D. Metformin reduces all-cause mortality Quelle est parmi les propositions suivantes celle qui est exacte concemant le traitement de la Surcharge graisseuse non alcoolique da foie? ‘A. Metformine (Glucophage) reduit le taux des transaminases hepatiques et ameliore les trouvailles a echographie B, Metformine reduit la possibilite de la progression vers la cirrhose C. Metformine reduit la mortalite cardiovasculaire D. Melformine reduit toute cause de mortalite i Gaal ens UN ul Ge gt Se ot mo ya la) (CBS) Cae rgd LA i Suny GOH Joule lyse Spb Ges | gg) oy Ge eyed LB IBD Jone Ge di ta UC ihe BG Ley, CA lal US Gan pital Gai D 35 [35 year-old female patient, smoking, with hypertension | gli gam Lie T° Bye Lime and a personal medical history of right leg phlebitis She | gesnyy alg ye Ana ye dyuey hee asks for the best means of contraception, What method ‘nab dade Local Ua do you recommend? eed 3 tisk oo te urcomnd ena oe B-Female condom, aes C- IUD (Intra Uterine Device) oh bs B D. Progesterone implant es E-Vaginal Ring aja sge hae -D tue tile OE Femme de 35 ans, tabagique, hypertendue avec antécédents de phiébites des MI se présente pour contraception, Quel moyen li recommandez-vous ? A. Pilule B. Préservatif féminin C Sterlet D, Implant de progestérone E._Anneau vaginal 96 | Which of the following matemal conditions isa Shae ga SI yA) ea Ga gi contraindication to breast feeding? A. Status post reduction mammography B, Hepatitis C carrier status C. Mastitis: D. Active herpes lesions on the breast Quelle est parmi les conditions maternelles suivantes celle qui est une contraindication a V'allaitement 7 A. Statut de mamagraphie aprés une chirurgie de reduction B. Statut de porteur d’ Hepatite C C. Mastite D. Lesions d’herpes active au niveau du sein fee Guba! eb gS ype AIG A, ¥ jaya GS isd Gea Ula. B st he path cle AA Gu ws SD 97 [A S-yesr-old girl presents for acute pain for Zhowrs in SD SEL pe as the left iliac fossa with nausea. An ultrasound done SS pad BS a J gel showed an enlarged left ovary. vasad gb adele Sas Spall Bb Oe U2d What is your first diagnosis {5,8 oath gs Legal Aca divertiulitis wputl A -An ectopic pregnancy © Coronet tne ovary euic kde B D-Acute appendicitis with intestinal malrotation crane gC E-A neuroblastoma pe BD eo ea eel! DD eee tips EB Lne fille agée de 9 ans se présente pour douleur aigué depuis 2 heures @ la fosse iliaque gauche avec nausées. Une Echographie faite a montré un ovaire gauche augmenté de volume Quel est votre promier diagnostic Ac Une diverticutite B- Une grossesse extra-utérine C+ Une torsion eV ovaire D- Une appendicite aigué sur malrotation intestinale £- Un neuroblastome 98 | Visual field defect involving loss of vision in the lateral Cast Gy Gal Gal oad Dd half of both eyes is due to AS Ge pula Goat 5 5. gla A. Lesion involving the temporal lobe fd dys gall B. Lesion involving the occipital lobe (pee gail Canes TA C. Lesion of the parietal lobe id od cys IB D. Pituitary tumor “jad ath sist C E. Bilateral optic neuritis $ we} cS pos E Un déficit de champ visuel interessant les champs externes des deux yeux est du a Lésion du lobe temporal B. Lésion du lobe occipital ©. Lésion du lobe pariétal D. Tumeur hypophysaire E__Nevrite optique bilatérale alla RE ge Gone lg {90 ‘A38 year old female patient, PIPOAO, obese, currently pregnant at 36weeks of amenorrhea + 2d, twin pregnancy, has a considerable increase the last two days of her leg edema with an intense epigastric pain in bar What would you do? ‘A-Reassure the patient: the leg eseuna is normal during pregnancy and prescribe Gastrimut 1B. Make an hemoglucotest C-Measure BP, proteinuria, CBC, complete liver function, fibrinogen and an urgent C-section D-Vaginal delivery E- Prescribe diuretics Patiente de 38 aos, GIPOAO, obese, actuellement enceinte a 36 semaines d’amsnorthée + 35, grossesse ‘gémellaire, présente depuis 2 jours une augmentation considérable de ses EEdemes des Membres inferieurs(OM1) accompagnée d’une douleur épigastrique en barre intense, Quelle est la démarche & suivre? ‘A. Rassurer la patiente «les OMI c'est normal durant la grossesse et prescrire un pansement gastrique Faire une glycémie Mesuret TA, faire une protéinurie, avec FNS, bilan hépatique complet, fibrinogene et césariser en urgence D. Accouchement voie basse E__Preserire des diurstiques ae <4, PIPOAD stale TAG pac yal Ub cas lg con ye + he gal PR ale, eet Sa Lea Hay gf Maga 5 S Lash el ayo Laat op pl fang Ia g0 8: Lana ila AL Se jill heyy and DL pd Su gasi thal Sach Gaypit Uy dial GC hye 36 Ray lS pak hs Ba peed aby el yay steal ‘ple Ages ily Cl jae Gs mo 700 You receive at your consultation a pregnant woman in the ath month for a urinary tract infection without fever. The urine culture grows with E, Coli Multi sensitive. Which of these antibiotics do you prescribe? ASpiramycin B-Norfloxacin C-Ceftazidine D.Ciprofloxacin E-Amoxicillin Vous recevez a 1a consultation une femme enceinte au dene mois pour une infection urinaire sans fidvre. LECBU pousse en E. Coli multi sensible. Lequel de ces antibiotiques but prescrivez-vous ? Spiramycine Norfloxacine Ceftazidime Ciprofioxacine Amoxicilline mON p> PIM oot gf dae yl Spee el abe Gok tly Go SS Oe ols SB IS USA ye Ll ED Tt | A'82 years old woman under oral anticoagulation for ai, shaw gil Lie AY ape Ty atrial fibrillation presenting with Diffuse abdominal GSE pay yea wall lla a gad pain for two hours with bloody diarrhea. First Sel en ose Se phn la Sh ye diagnosis: pags gaa A. Gastroenteritis, oe at A B. Hemorrhoids yee 33 C. Intestinal obstruction ate D, Mesenteric ischemia gore deat CS E. Irritable bowel sin 3D wate Ws ERE Femme de 82 ans sous anticoagulation orale pour fibrillation auriculaire. Douleur abdominale diffuse depuis 2 heures violante avec diarrhée sanguinolente Premier diagnostic A. Gastroenterite B, Hémorroiddes C. Occlusion intestinale ©. Ischémie mésentérique E, Colon irritabie 102 | An elderly patient with hypertension who presents with | hee E45) ge glu od GS ef Ue a lateral medullary stroke would be expected to have all of the following EXCEPT Contralateral Babinski sign Ipsilateral Horner's sign Ipsilateral cerebellar abnormality Vertigo Loss of pain and temperature sensation on contralateral side of the body and ipsilateral side of face moas> Un patient age ayant hypertension qui se presente pour tun accident vasculaire bulbaire, vous attendez a avoir tout ce qui suit saat Signe de Babinski controlateral Signe de Horner ipsilateral Une anomalie ipsilateral cerebelleuse Vertige Une perte une sensation de la temperature et de la douleur du cote controlateral du corpset du cote ipsilateral de la face mone> Le JS Bsns ale Aas GS Cl tae be gly Ui hall Sty ROME A a Gad ys ADE Rv ertepeterse-nel yD Gilad slaty ANY Glas) Gludi E pol thal aig pall Ji 703 Which of the following inflammatory conditions is most likely to lead to a pyogenic tiver abscess? A. Cholangitis/ cholecystitis B. Diverticulitis C. Endocarditis D, Inflammatory bowel disease E, Perforated appendix Quelle maladie inflammatoire est 1x plus probablement associée Avec la formation d’abcés hépatiques? A. Angiocholite/cholécystite B, Diverticulite C. Endocardite D. Maladie inflammatoire de lintestin E._Appendicite pertorée JS Bp GS Gay! LG Gt 8 gh eth ot gas Si La eas Spe lye al os Cyt A, coat lel B wa Gas Gh cdat uss ele -D igaidy E 104” | Which of the following infectious agents is most highly OSS BLS AI NT La Ge gt aezocnted wih anal carcinoma? Mineo Se 8 B. Epstein Barr virus reese 8 C. Herpes simplex virus we ee D. Human papitloma virus cy I ae E. Human herpes virus 8 PN ll od od -D 8 goed Od Gy -E Lequel des virus suivants est le plus associé & un carcinome anal? A. Adenovirus B. Epstein Barr virus C. Herpes simplex virus D. Human papilloma vires E._Human herpes virus 8 05 _|/A 30 year-old female presenis with 2 days of headacke, | fever, seizure and confusion, On exam there is weakiess on one side of the body and stiff neck. The CSF shows 20 wae baa s WEC's, 0 RBCs with normal sugar and protein, The | MO" A SSS SA es cond MRI of brain reveals a mass effect with edema in the yan cS te slag oS temporal lobe, The most likely diagnosis is: Foe eee ~ A. Herpes simplex encephalitis ed oat oes sata ay B, HIV with brain lymphoma ee ee post C. Septic embolus secondary to endocarditis FAST mt geval nit D. Brain tumor : 2 eAaly hey ela eles Kel A Une femme agee de 3 ans se presente pour une Sapa cephalee, fievre, convulsion et confusion depuis 2 jours. abled got ey GH HIV.B | A examen il existe une faiblesse du corps unilateral et a GA gay us asl eC une raideur de la nuque. Le liquide cephalorachiciien wel paxD montre 20 globules blanes et 10 globules rouges avec tune glycorachie et une proteinorachie normale. L'TRM cerebral revele un effet de masse avec cedeme au niveau du lobe temporal. Le diagnostic le plus probable est A. Encephalite a herpes simple B. HIV avec lymphome cerebral C_ Embolie septique secondaire a une eondocardite L D. Tumeur cerebrale B48 106] Which of the following findings is the strongest a indication for imaging in patients with acute low back pain? A. Failure to respond to conservative therapy after two to three weeks 2B ely & GI gl il G8 ae a oe Ustle gaa ys ae oe cae wld gpd ts UT JOA | hea ato B, Bilateral leg weakness C. Sciatica in one leg, Gh ste D. Positive result on leg raising Bashy Jay gd LU GC dat Gb te Ga GED ‘Quelle est parmi les suivantes celle qui est l'indication la plus forte pour investiguer en imagerie un patient ayant tune douleur lombaire basse ? A. Echec de la réponse au traitement conservateur aprés 2 0u 3 semaines B. Faiblesse bilaterale des deux jambes CC. Sciaticalgie au niveau de la jambe D._Resultat positif en soulevant la jambe 107 | In the evaluation of dizziness, a true statement is: ‘A. Acute vestibular neuritis is the most common cause for dizziness B. Once vertigo is diagnosed the next step is to determine whether the cause is peripheral or BS gall eat BS ad ga ct yl Ua aw osha ol B peat 3 poses tll JS 1 Ah ee C Labyrinthitis is similar to acute vestibular sy ett ach So c neuronitis sonal ey a te ol D. History usually does not reveal the diagnosis aaron | SUSY fale Kase pls pal 9 D Dans evaluation d’un vertige, quelle est Ja proposition cask ie exacte A La névrite vestibulaire aigue est la cause la plus fréquente du vertige B. Quand un vertige est diagnostiqué I'étape suivante est de déterminer si la cause est centrale ou périphérique C.La labytinthite est identique a la neuronite vestibulaire aigue __| D.Lhistoixe ne révéle pas habituellement le diagnostic 8 ‘A 89-year-old woman has been hypertensive for the past 5 years. She is on hydrochlorathiazide 50 mg daily. On examination, her blood pressure is 130/80 mm ig, ‘The apex cannot be felt. What is the most sensitive investigation to assess left ventricular hypertrophy in this patient? Plain chest X-ray postero-anterior view Plain chest X-ray lateral view Resting electrocardiogram Exercise electrocardiogram Transthoracic echocardiography mon e> Une fernme age de 53 ans a été hypertendue Durant les 5 dernigres années. Elle est sous hydrachlorothiazide 50mg par jour. A ’examen, sa pression artérielle est de 130/80mmig. On rarrive pas palper apex. Quelle est Investigation la plus sensitive pour évaluer Vhypertrophie ventriculaire gauche cher. cette patiente? A. Une radiographie simple du thorax incidence antéropostérieur B._ Une radiographie simple du thorax en lateral CC. Blectrocardiogramme au repos D. Electrocardiogramme a l'effort E__Echocardiographie transthoracique Thine GUE) Ge Gis We OF La pe ala Ges REE yee Sas chee Oe pl ie J veal oe gab "Ley pp gene 3 Lh) hf pest Sayan Sts ead laa og pd $ Ria ab okt sie pol Sula Ay = a ste Real ype cael peal! Guile Ssle GANG ype B Bat ae Shes A das .C any Gola gS lb bbs D pall Gey all gave babs E 709 Which of the following tisk factors predicts the highest risk of developing breast cancer? A. Mother with breast cancer 8, Nulliparity C. BRCA-1 or BRCA2 mulation D. Age >55 B. Biopsiy-proven lobular carcinoma in situ Quel est parmi les factears de risqué suivants celui qui predit le risque le plus haut de developer un cancer du sein Une mere ayant un cancer du sein Nulliparite Mutation du BRCA-1 ou BRCA-2 Age superiour a 55 ans Carcinome lobulaire in situ prouve par biopsie mone: DST RS Ts IR LGD oe gl © ll He pe etd 8 Bee git dA Slnay) gle fal pe -B BRCA2 ,BRCA-L sib .C £9 a8 ye -D E Be Sar Oe cl tt the 170 ‘A 67-year-old woman presents with acute severe back pain On examination she looks pale. There is no localized tenderness of the spine. Bone profile showed, the following: Calcium — 2.9 mmol/l (22-26) Phosphate 22mmol/I _ (0.8-1.2) Alkaline phosphatase 126 iu/I_—_ (50-150) Which ONE of the following is the SINGLE most likely underlying diagnosis? Hyperparathyroidism Metastases Multiple myeloma ‘Osteoporosis Sarcoidosis. moos> Une fernme agée de 67 ans se présente avec une douleur sévere et aigue au niveau du dos. A examen elle parait pale I n’existe pas de douleur localiste av niveau de la colonne vertébrale. Le profile osseux montre Calcium — 29mmol/1 (2.22.6) Phosphate 2.2mmol/l__(08-1.2) Phosphatase Alkaline 126iu/t (60-150) Quel est parmi les suivants le seul diagnostic le plus probable ? A. Hyperparathyroidie B. Metastases C. Myelome multiple D. Osteoporose E._Sarcoidose gab a De gl Le ta po sll Say Y gaan ell Qos Gay nals ad Sel patel) aga gill ce sie OU 1 lt (reer) (Vetch) jan ile oY les WSs) Aoi VT lal ta Qe JS a Gas gs Gl On gl Puss Goh Be bls kA ae. onde Gi a9 -C Abe 4Lts.D aS E ® 55-year-old woman who was diagnosed 10 have colonic diverticula on a computed tomography. She is ‘cutrently asymptomatic and is presenting to your clinic for counseling. ‘A. More than half of patients with diverticula, develop GI malignancy B. Colonic diverticula result in bleeding in the stool C. She should increase the fiber content of her diet D. Frophylactic antibiotics should be given to prevent diverticulitis Une femme agée de 55 ans qui a ete diagnostiquee comme ayant une diverticulose chronique sur le CTsean est actueliment asymptomatigue et se presente a votre clinique pour un conseil ‘A. Plus que 50% des patients qui ont des diverticules, developpent une malignite du tractus gastrointestinal B_ Les diverticutes coliques donnent une hémortagie dans les selles C. Bile doit augmenter le contenu de fibre dans sa diette D. Les antibiotiques prophylactiques doivent etre donner pour eviter la diverticulite Woakl Sy, Ue 8 we ald Pct pail Had I ey Ge Babel Gas ay Galyel Ge Gils Y sc Gall + ales Gh gem yl a oe SLA Koes Ras pos) ppd Com eS AERP LD goal or OLB Shoal Aa pb GUT fine bn i GI aC lee co gl yall Chala ele! gD cot eat B49 "2 The drug of choice tor an elderly patient who has Akinesia (slowness in his motor activity), a resting, tremor and cogwheel rigidity is Aspirin anticoagulation Anti-cholinergic agents Dopamine Dopamine Agonists L-Dopa mons> Quel est parmi les suivants, le medicament de choix pour un patient age qui a une akinesie (raientissement de son activite motrice), un tremblement au repos et une rigidite a la marche ‘A. Anticoagulation a laspirine B. Agents anticholinergiques ©. Dopamine D. Agonistes de la dopamine E__L-Dopa ad me as Co ad ol gh Gees Sal Se Ate AS a he ares cel, algSll Saline Gul eB onby cogs a. Uy GE iz} ‘A 20 year-old previously healthy man presents to the emergency room with a history of fever, headaches and dry cough of 3 days duration. Physical examination was only remarkable for a temperature of 39.5°C. Blood pressure, respiratory rate and lung exam were normal, Chest X-Ray revealed right and left lower lobe infiltrates. Your options for treatment include all the following except A. Oral clarithromycin B. Oral doxycycline C. Oral erythromycin D. Oral levofloxacin E, Intravenous ceftriaxone Un horame age de 20 ans qui est auparavant en bonne sante se presente aux urgences avec une histoire de fievre, cephalee et toux seche et ceci depuis 3 jours. L’examen physique n'est remarquable que pour une temperature de 39.5YC.Ja pression arterielle, la frequence respiratoire et l'examen des poumons sont normaux. La radiegraphie du thorax revele des infiltrats lobaites droite et gauche. Votre option therapeutique inclue tout ce qui suit sauf Clarithromycin oral Doxycycline oral Erythromycine orale Levofloxacine oral Ceftriaxone ex. intraveineux mone> Wie any Gas "Ue Tes we de] F Me le yes ah eh pee BN GA Saas Eldee a Ly ce od Basak Le EGS gyal Gand BS Dy all Jane spall bie THLE aay Relea jacal tm Ge Sa Ath Sey Oat lid Gail gb Chale Phe gh SS gM LS Gen] pet Cae A, 8 oe oS eB pa es BIC ag 94h Cpl liga. sy Oath he 14 In a patient presenting with pharyngitis, which of the following is not consistent with Streptococcus pyogenes, infection? Fever Tonsillar exudates Lymphadenopathy Odynophagia Hoarseness mone> Un patient qui se présente avec une pharyngite, quelle est parmi les suivantes celle qui ne va pas avec une infection a streptocoque pyogene ? A, Fitvre B. Exsudat au niveau des amygdales C. Lymphadenopathie 1D. Odynophagie E__Enrouement de la voix Oe gh ep ald yal Glee Gay ae | By 88 GB Gale de onl a Sl © kia Sid tA TIS] Which of the following joint is never affected by gout? |. a Ta) Guay Y AID Gala Gm Gl A. Metatarso-phalangeal joint ¢ B. Knee joint ead pha Junta A C. Hip joint 2S) uate B D. Interphalangeal joints Bog dente E. Intervertebral joints you gate Saas D Quelle articulation n’est jamais atteinte dans la goutte ? Sa deal) JE ‘A. Articulation métatarso-phalangienne B. Genou Hanche D. Articulations intesphalangiernes E._Articulations intervertébrales 118 Which of the following is the most common form of cancer affecting the skin? ‘A. Basal cell carcizoma B. Squamous cell carcinoma C. Malignant melanoma D. Cutaneous T cell lymphoma E, Metastatic lung cancer Quelle est parmi les suivantes la forme la plus frequente dlucancer atteignant la peau? Carcinome baso cellulaire Carcinome spino cellulaire Melanome malin Lymphome cutane a cellule T Metastase du cancer pulmonaire nROAS> Th ple ye FEN EM pw Gh gl fais) Rec DT hye A i a gh B Sok pide pa -C Suh T 6 Lind D eS, hye E [117 TA biliary-enteric fistula most frequently connects the gallbladder with the A. Duodenum B. Jejunum ©. Stomach D. Hepatic flexure of the colon Une fistule entero biliaire relie le plus frequemment la vesicule biliaire au ‘A. Duodenum B. Jejunum C. Estomac D. Angle colique hepatique BD Be A pd pe Pad ote i OA sual B andl gal iia SD 118 | Which one of the following antibiotics would be an SB Sep GI Gad lace Go gt sppropriate single-dose treatment for patients with alll pea pba dade de pS Qu gonorrhea wah Dhnd e gsley ‘A. Penicillin ht B. Tetracycline ok SB C. Trimethoprim-sulfamethoxazole ~ D. Ceftriaxone nna Gin Se Quel est parmi les antibiotiques suivants celui qui est un traitement approprie d’une dose unique chez les patients ayant une gonorrhee ? A. Penicilline B. Tetracycline C. Trimethoprim-sulfamethoxazole D_ Ceftriaxone 149 | Which of the following is the most common cause of male factor infertility? jaculatory duct obstruction Primary hypogonadism Secondary hypogonadism ‘Testicular tumor Prader-willi syndrome mon g> Quel est parmi les suivants la cause la plus fréquente du facteur d!infertilité chez les hommes? Obstruction du canal ejaculatoire Hypogonadisme primaire Hypogonadisme secondaire ‘Tumeur testiculaire Syndrome de Prader-willi mIog> De ys SH al po GD Ge gt # Sia se ‘i GU SLA AMI Kal 208) 5B pA La 2h aC Anal of po9D gels ~ A be Di 720 Within these antibiotics, which one doesn’t have effect on the Bacteroides fragilis? A. Clindamycin 3B. Cefoxitin C. Aztreonam D, Chloramphenicol E.Imipenem ‘Quel est parmi ces antibiotiques, celui qui n’a aucune action sur le Bacteroides fragilis ? A- Clindamycine Be Cefoxitine © Aztréonam D- Chloramphenicol E-_Imipéneme ead pila tad Glad & Gl Aa Cl pelle ceeds A oS se ApS! J sSyiutal IS Gel Hoae TH ‘A @S-year-old man presents with the complaint of hard swelling in his distal inter-phalangeal joints in both hands for few months. On physical exam the lesions are hard and not tender, A true statement about this finding A. This man has osteoarthritis B. Carry the Lachman’s test to confirm your diagnosis c D. Request a bone densitometry E. X-ray of the hands is advised at this point Un homme agé de 65 ans se plaint de gonflement dur au niveau des articulations interphalangiennes distales au niveau des deux mains et ceci depuis plusiours mois. A examen les Jesions sont dures mais indolore. Quelle est a proposition exacte concernant ceci A. Cet homme a wne osteoarthrite B. Faire le test de Lachman pour confirmes votre diagnostic C. Demander une densitometric osseuse D. Une radiographie des deux mains est & conseiller a ce niveau ee apll C1 Go Sy Ule 1 ope ny oe «pel bs0 Sa ay ald Jeol hig “lb GUS) ok Uh gy pall Gaal op BED 65g) Ragone had) Aaya cay e tle Sha Oe ln Jat LOA, ASS Gat sg JS Goad lal B vasa phat GES Li opal all C Wad oh 122 ‘A 6-hour-old newborn exhibits tachypnea and dyspnea and vomits repeatedly. The body temperature of the newbom is normal. The physical examination reveals a tympanic resonance over the left side of the chest-, no respiratory sounds are audible over this side. The surface of the abdomen is concave. What is the most likely cause of this condition? A. congenital valvular disease B. neonatal sepsis C left-sided pneumonia D. a diaphragmatic hernia on the left side E, pneumothorax Un nouveau-né agé de 6 heures présente une tachypnee ‘et une dyspnée et vomit a repetition. La temperature de son corps est normale, L’examen physique révele une resonance tympanique au niveau de la partie gauche du thorax, les murmures vesiculaire ne sont pas audibles & co niveau. L’abdomen est ceneave. Quelle est la cause la plus probable de ce tableau ? ‘A. Maladie congenital valvulatre B.Un sepsis neonatal C-Une pneumonie gauche D. Une hernie diaphrogmatique gauche E,Pneumothorax Tong RS ga ELL ao Dy Cae Hee Jos a Si oll oil O82 gael ceil geil Arath sa Yigal ge aS Coal Ga J Ba ie 8 Xe ges i I pel ies J at ye Ls abe ob ch © ad aigh alma ooh | as ot B aft Calg 1) 013.6 Sef lal gla GSD seal chy ful E 723 |The most important predictor of survival after surgery for | sad af le el Gal ASW 525 gf advanced ovarian cancer is 1 yh pile ape Gb ad hye Hae ay A. Presence of ascites clinted aya A B. Histologic grade 51 aa B C. Type of chemotherapy used Dales Cc D. Volume of tumor remaining after surgery dal ob we wiae D Le facteur le plus important qui predit la survie apres chirurgie pour cancer ovarien est A. Présence d'une ascite B. Grade histologique C. Type de chimiathérapic utilisé D. Volume de la tumeur tésiduelle aprés la chirurgie | 124 | Which of the following is the recommended initial TY Fy AS las IS Ge gt therapy for viral pleurisy? 8 pes ll Guall A. Opiate analgesics sydd lh A B. Colchicine AB CC. Nonsteroidal ant-inflammatory drugs siaiyge ye Ugo Dee D. Acetaminophen * . CA pte | Quelle est parmi les suivantes le traitement initial recommandé en cas de pleurésie virale ? A. Analgésie aux opiaces B. Colchicine CC. Antiinflammatoire n D._Acetaminophene n steroidien. [725 Whick of the following entities is most associated with) severe pneumococcal sepsis? ‘A. Multiple myeloma B. Acute myelogenous leukemia CC. Non-Hodgkin lymphoma 1. Small cell carcinoma of the lung E, Gastric carcinoma Laquelle des conditions suivantes est le plus classiquement associe avec une septicémie & pneumocoques severe ? A. Myélome multiple B, Leucémie myéloide aigue C. Lymphome non-Hodgkinien D. Carcinome pulmonaire a petites cellules E,_Carcinome gastrique WP5 SIE eB Gl g! wees oA 2 ga pe ala Used oSaaet EARN She LS Hea ase ls B mt p | E | 126 | Among patients with chronic kidney disease, metformin | pal clo ya plas ON yay) Gua Oo (Glucophage) generally is recommended for which one phn AID Ce papal a GV sce of the following groups? © i {A Patients with chronic heat file (esas) nosis B. Men with serum creatinine level of 1.5mg/dl or Cede ih aaa On Osis oo mks Lele > Seal 8S p40 Jay B C. Patients older than 80 years ae D. Patients with severe infection We soe oh greet Sanat SH Ge Oy oye Parmi les patients ayant une maladie rénale chronique, Ta metformine (glucophage) généralement est recommandée pout lequel des groupes suivants ? A. Patients ayant une insuffisance cardiaque chronique B. Les hommes ayant un taux de eréatinine égale a 1,5mg/dl ou inférieur C. Les patients ages de plus que 80 ans D. Patient qui présente une infection severe 127 [A patient is brought to the emergency department cus ce jyal 42 G2 vale >asl because of a suicidal attempt by ingesting massive doses of aspirin. Which of the following would be helpful in managing this patient? ‘A. Acetaminophen B. Amphetamines C. N-Acetylcysteine D. Phenobarbital E. Sodium bicarbonate Un patient est amené aux urgences apres une ingestion suicidaire massive d’aspirine. Quel serait utile dans son traitement? Paracétamoi Amphetamines N-acétyleysteine Phenobarbital Bicarbonate de soude Imona> lege IY Geo Ue asi! sla DF Bae II Ge sh ran Se 5 pas Seas ys gb sey spain outa B cpStane UNC Jseeed D penal Sei gE 8 ‘At her week postpartum visit, a 21-year-old woman, gravida 1, para 1, tells her physician that she hasa pinkish vaginal discharge that has persisted since her delivery, although it is decreasing in amount. On. physical examination, the uterus is fully involuted, and there are no adnexal masses, Which of the following is the most appropriate next step in management? Administer ampicillin Order a quantitative beta-HCG test Order a serum prolactin measurement Reassure the patient that this is normal Schedule a dilatation and curettage mongs> ‘Asa visite 6 semaine en post partum, une femme dgée de 21 ans, gravida 1, para 1, raconte a son medecin qu'elle a un écoulement vaginal violace qui persiste depuis son accouchement, et qui diminue en quantité. 4 Vexamen physique, I'utérus est completement en involution, et il n’existe pas de masse adnexielle Quelle ‘est parmi les suiventes I'étape la plus appropriée dans la conduite a tenir ? ‘A.Administrer de l'ampicilline B,Demander un test quantitaf de beta-HCG C Demander un taux de prolactine sérigue D.Rassurer la patiente que tout est normal EPrévoir une dilatation et un curetage Spd GV gl GID Gola g gal 671Gb hye IM dala CLL FY Ua a oles Gas Oo eles Lally Lele Ce al Oe eS Se pe OB gaged anil on ab YS al | gh Rabe BSI aga Vy gan sul alle A Gb LD GIGD FS a ID Oe geal Cdeaual Usel A, BCG be Saat sd B Sead (iS gH 4 plas Lael C apt AN Nhe ob teas yall ab. Ly ge sil Jae Yt E 728 The most serious complication of casting and splinting ‘Thermal injuries to the skin Compartmental syndrome Bacterial and fungal infections Pruritic dermatitis, Joint stiffness due to immobilization moOg> La complication a plus serieuse du platre et de la mise en place d’attelle est ‘A. Traumatisme thermique au niveau de la peau B, Syndrome du compartiment C. Infection bacterienne et fongique D. Dermatite prurigineuse E._Roideur de articulation due a immobilisation — ae BS ela | Sk eels al aD Aja ULL Jad 4 ea B Betas oe ou la gals iy. op gil Gas) GLE 730 Ina young man presenting for dyspnea, physical exam reveals near absent lower extremities pulses. The most likely diagnosis is: A. Arteriosclerosis B, Coarctation of the aorta C. Takayasu aortitis D. Cocaine use E, Ergotamine derivative use for migraine Chez un jeune home se présentant pour dyspnée, Vexamen physique révele I'absence de pouls aux membres inférieurs. Le diagnostic le plus probable est Artériosclérose Coarctation de 'aorte Aortite de Takayasu Utilisation de cocaine itilisation de dérivés de lergotamine pour migraine mOnS> Oe gly AEs papal Gem Oo Se ytd GLY Gad hs G2 sd Spe bgt Maa FSU essa bes A A! Spas B “est jel tg -C oS ND gboeall Guliye JW citi pss LE ail 73 Which one of the following findings suggests foreign body aspiration? A. Wheezing that occurs bilaterally B. Chronically ill appearance C. Sudden onset of wheezing D. Nighttime coughing Quelle est parmi les suivantes celle qui suggere une aspiration de corps etranger? ‘A. Wheezing qui se produit des deux cotes B. Malade qui parait malade d'une fagon chronique C. Début soudain de wheezing D. Toux nocturne chal aaeee. 132 Which one of the following is an adverse effect of alpha blockers? A, increased incidence of prostate cancer B, Increased incidence of high-grade prostate cancer C. Orthostatic hypotension D. Ejaculatory dysfunction Laguelle parmi les suivantes est effet secondaire des aipha bloqueurs ? A. Incidence augmentée de cancer prostatique B. Incidence augmentée de cancer prostatique de haut grade C. Hypotension orthostatique D._Dysfonctionnement éjaculatoire WSR eS Tp Go gl] heyy le po Ge Sas CEA as bps al Aas CEB ile tats esl 855 lind. Ga Bb, 8 LD 733 Transmission of perinatal infection is done in several ways (all propositions are true except one) ‘A-Way transcordonale; road ascending through fetal membranes ruptured prematurely; C- Inhalation to the fetus of maternal genital secretions soiled; D-The use of forceps for fetal extraction: E-At a neonatal resuscitation with possible insertion of prostheses, La transmission de ‘infection périnatale se fait selon plusiears voies (toutes les propositions sont justes saut une) A- Voie transcordonale ; B- Voie ascendante a travers des membranes foetales rompues prématurément ; C- Inhalation par le foetus de sécrétions genitales maternelles souiliées ; D- Llutilisation de forceps pour une extraction fortale ; E- Lors d'une réanimation néonatale avec insertion eéventuelle de prothéses. BY DA sl CY) J Sans Uden AID Gall JS) Gy saw eet a oe 4S ye acl Gee daualil tM yaad Stal Ny Ga Gite C opel Gad bial plas! .D. GN Dei) case hal E a oeal (be A B Tea Several patients present at your clinic simultaneously, with symptoms occurring 2-3 hours after a meal. Their complaints include malaise, nausea and vomiting although they have no fever. Which of the following foad-poisonings is the likely cause of this condition? Botulism Salmonellosis Staphylococcal food poisoning Clostridium perfringens food poisoning Campylobacter monges> Plusieurs patients se présentent en méme temps a votre clinique, pour des symptomes qui sont apparus deux a trois heures aprés un repas, Leurs plaintes comprennent des malaises, des nausées et des vomissements mais pas de fievre. Quelle est parmi les intoxications alimentaires suivantes celle qui est la cause de cette présentation ? ‘A. Botulisme B, Salmonellose C. Intoxication alimentaire a staphylocoque D._ Intoxication alimentaire a clostridium perfringens E._Campylobactere isin Se pe yee tials pa Gel TAY aay Cla Gil eh Ge Se ced SAL SS Geet ing ISO Soe toms pie Ga De ASL hts ISS Bag AI AIST Le Go TLD oS) Capel Fats pat A Miyalall Be Agi CA jay gS peal Fabled Cady Sill peat D. ke ull B38 135 | All the following diseases may be associated with diarrhea except A. Addison's disease B. Hyperparathyroidism C. Hyperthyroidism BD. Medullary thyroid carcinoma E, Pheochromocytoma Laquelle des maladies suivantes n'est pas associée a une diarthée? A. Maladie d’ Addison B. Hyperparathyroidie . Hyperthyroidie D. Carcinome médullaire de la thyroide E,__Phéochromocytome eal BLS Go! Se lh Gat ANI US Shel pant els A sss ess B SM bby .C SP wpb DS Bes E 136 | Which of the following is considered as a Pele Sibi as oie Ba oe gl contraindication for surgery in lung cancer? ib lb A. volverent of the diaphragm pal yall ca oA B. Extension to the chest wall Ian d sac © nvesion othe superior vera ava eget D. Invasion of the pericardium nD E. Allof the above opt chacss fase el Syl USE Laquelle est une contraindication chirurgicale dans le cancer du poumon? ‘A. Invasion du diaphragme B. Extension ala paroi thoracique C. Invasion de la veine cave supérieure D. Invasion du péricasde E. Toutes 737 | A 60 year old female patient presents to your clinic with right knee pain which you diagnosed as osteoarthritis, Which one of the following treatment modalities is the most effective? A. Arthroscopic surgery B. Steroid injections C. Knee braces D. Chondroitin supplements Un femme agee de 60 ans se presente a votre clinique avec une douleur du genou droit vous diagnostiquez comme etant une arthrose. Quelle est parmi les modalites des traitements suivants celle qui est la plus efficace ? A. Chirurgie arthroscopique B. Injection steroidienne C. Contention du genow D. Supplement de chrondroitine all) Gh pan “Lle Te W pe ald sas giell ASI al Se Sty 2 A Kall Ga Gl aa ad 2 ks 3S oe elon ghaie hc goby oad GIB 4S jh de Siu) cles 138 Which of these skin lesions is specific for lupas, erythematosus? Av Raynaud’s Syndrome B- Livedo - Usticaria D- Necrotic purpura E: Discoid lupus Laquelle de ces lésions detmatologiques est spécifique du lupus érythémateux disséminé ? ‘A- Syndrome de Raynaud Be Livédo C Unticaire D- Purpura nécrotique E+ Lupus discoide 425 jas WD Ral CALAYT oe gt | ayaa Ga dpb De A as mone 139 | Splenomegaly is present in: Bush LES Ss ‘A. Aplastic anemia ent ga A B. Multiple myeloma sie Bey B C. Primary myelofibrosis ub < D. Thalassemia minor coke bees .D B. Sickle cell anemia in the adult BS) eu it OE La splénomégalie est présente dans ‘A, Anémie aplastique B.Myélome multiple C.Myélofibrose primitive D, Thalassémie mineure E. Andémie falciforme de l'adulte 740 Which of the following drugs is the first-line teatment for post-partum hemorrhage? A. Oxytocin 8. Methergonovine ©, Carboprost D. Misoprostol ‘Quel est parmi les medicaments suivants celui qui est le premier traitement d'une hemorragie en post-partum A. Oxytocin B. Methergonovine Carboprost D._Misoprostol IMT AILS gs Gl GM oe gh 130g ay sae LA JsaespseeD vat ‘A 40-year-old woman who presents fo your dinic with & bad headache since two days. It began suddenly with a severe headache, There are no neurological signs noted You tell her that you need to refer her for an investigation in the hospital, She asks you what investigation will be done, Which ONE of the following is the SINGLE BEST diagnostic investigation? A.CT scan B.EEG C. Lumbar puncture T1 D. MRI E, Skull X Ray Une femme agée de 40 ans se presente a votre Clinique pour une céphalée intense depais 2 jours. Et quia commence subitement par une céphalée severe. I existe pas de signe neurologique. Vous tui dites que vous avez besoin de la referer pour investigation dans tun hopital, Elle vous demande quelle sorte d'investigation devrait éire faite. Quelle est parmi les suivantes la seule meilleure investigation diagnostique & faire ? A. CT scan DEEG C-Ponction lombaire au niveau de TL D. IRM E Radiographie du crane Ebel f+ seal Ge Adal yh Ebsee SS ated be By oye ys ie eee (Gintet gune GILL OSL Al ase haat Sal oe tall I Lea A atl go Lele OS wb as | GA eID Ge gh sod ie Saas Sa fae git ca A flasl de nS bubss B Thahb IC rubles gy gy p2i-D AWE po E 142 Which of the following disorders is an indication for lithium therapy? A. Panie attacks B. Schizophrenia €. Obsessive-compulsive disorder D. Bipolar disorder E Anxiety Parmi les situations suivantes, laquelle est indication pour un traitement au Lithium? ‘A. Attaques de panique B.Schizophrénie C.Désordre obsessive compulsif D. Syndrome tipolaire E.Anxigté une elt Ge ga UI SU hay Ge gt lls Ag weeks A feast pail B il lye Global oC lel giib a gal -D ai E 743 The most common differential diagnosis of myocardial infarction is: A. Pericarditis and pleuritis, B. Gastroesophageal disease such as reflux or esophageal motility disorders C. Pulmonary embolism D. Ruptured aortic aneurysm E, Herpes zoster Le diagnostic différentiel le plus fréquent d'un infarctus du myocarde est, A. Péricardite et pleurésie 8, Pathologie gastro-oesophagienne tel un reflux ou des troutbles moteurs cesphagiens ©. Embolie pulmonaire D. Rupture d’anévrysme de l’aorte E, Zona ERY eed S501 fll Gass "a gl Saad | hod ely A J ESE a gees B 190 AS ya SY pla c SS jae Kou oD E 144 | Lower endoscopic evaluation is recommended in which | pais Clb, 430) pm yl Sle pane Ge of the following groups of patients with iron deficiency | Shay ay a phd ea vglin Gla gs anemia with no obvious benign cause? ye oe a A Menunds) * Bere Ie Be B. Premenopausal women nas C. Postmenopausal woman ap gaheses nemres : D. Infants and children Sth Bad CE a yyy alg Ua a pL. Une evaluation endoscopique basse est recommandée AVN; DaeY.D dans quel groupe de patients parmi les suivants et ayant une anémie avec deficit en fer sans aucune cause évidente ? A. Homme agé moins de 50 ans B. Femme en premenopause C._ Femme en postmenopause D._Enfants et bebe 145 | What is the most common cause of death in peopie with Be ple pt al untreated hypertension? ‘A. Coronary disease B. Cerebrovascular accident ©. Renal failure D. Hypertensive encephalopathy E, Cancer La cause la plus fréquente de metalité chez des personnes ayant une hypertension non contraée ? ‘A, Maladie coronarienne B. Accident cérébrovasculaire . Insuffisance rénale 1D. Encéphalopathie hypertensive E. Cancer Oe gp hie UE) oe Gyles ald ghee pil ely pela gly Cala GS art ical Eli Ge gab gels lel sash he goe> in B-e 148 The most common cause of eosinophilia in developing countries is: A, Drug allergy B. Respiratory allergy C, Parasitosis D. Infection E. Asthma La cause la plus commune d’hyperéosinophilie dans les pays en voie de développement est ‘A- Allergic médicamenteuse B- Allergie respiratoire C- Parasitose D. Infection E-_Asthme Gee Se | AKA IAI Saya A a7 You receive at your consultation a 75 year-old woman who is vomiting since 3 days with signs of dehydration, Her arterial blood gases showed: pH 7.45, pCO2 = 45 mm Hg; Bicarbonates = 35 mmol /{. This is a ‘A: Respiratory alkalos B- Respiratory acidosis C: Metabolic alkalosis, D- Metabolic acidosis, E- Mined alkalosis Vous recevez & la consultation une femme de 75 ans pour des vornissements depuis 3 jours avec des signes de déshydratation, Ses gaz du sang montrent: pH= 7,45; pCO2= 45 mm Hg; Bicarbonates= 35 mmoles/l. Il s‘agit dune: Ac Alcalose respiratoire B+ Acidose respiratoire & Alcalose métabolique D- Acidose métabolique E Alcalose mixte Ge gist Ule VO Wpue alt | pF SE Seb has alae gh ese pH 7.45, pCO2 = 45 mm Hg; Bicarbonates = 35 mmol / 1. ole i ol er Oceans pad ysl ae oat sti ost Me moIAaS 788 The following chemicals may be associated with thrombocytopenia except ae Utypal chagiad gals ogee A. Alcohol sey B. Aspirin : C. Sulfonamides oe é D. Heparin $ E. Methotrexate che LS Sse E Les produits suivants peuvent etre associés a une thrombopénie sau A. Alcool B. Aspirine C. Sulfamides D. Héparine E._ Methotrexate (2 BID Asa yall SLB IO [749 ‘Type 2 diabetes patients should be screened for diabetic nephropathy during which of the following examinations A. Atinitial diagnosis B. At one year after diagnosis C. Atfive years after diagnosis D. Ar years.atter diagnosis, Des patients ayant un diabete de type 2 doivent etre depistés pour une nephropathie diabetique durant lequel des examens suivants ‘A. Au diagnostic initial B. Unan apres le diagnostic C. Sans aprds le diagnostic D._ apres le diagnostic ie ey FOR oe ae Seg Le G98 Died iF Phe eA a eA anil ge te yy 0 80B ep gle CI fae Gund yyy ats C oe gle OM gee pte yy ye aD Which of the following manifestations is not encountered in systemic rheumatoid disease? Pericarditis Amyloidosis Pancreatitis Vasculitis Carpal tunnel syndrome rons> ‘Quelle est parmi les manifestations suivanites celle qu'on né trouve pas dans la maladie rhumatoide systemique ? Pericacaite B. Amyloidose C. Panereatite Dd. E > Vasculite Syndvome du canal carpien 750 | Tm which of the following conditions is pleurocentecis | Jum) gonna tl GV Gn gl gt urgent? fs A-A giant lobar emphysema Bue yeni A B-A cystic adenomatoid malformation of the Ib gaSget py Rien B lung z CA right diaphragmatic hernia of oe o ae a 5 D-A left diaphragmatic hereia Tee ee E-A tension pneumothorax Beal eal chil E Dans laquelle des pathologies suivantes, a ponction pleurale est urgente ? ‘A- Un emphyséme lobaire géant B- Une malformation adénomatotde kystique du poumon C. Une hemie diaphragiatique droite D- Une hernie diaphragmatique gauche E- Un preumothorax suffocant - Ts AILS We Gal ALA Ge fg deall yg Sod ot SA oil chad B A Sal Hys.C ghey ga Pel Gi Din E 52 ‘A 2é-year-old pregnant women in her last trimester sustained an anikie sprain. She reports severe pain, 8/10. She was able to walk 10 steps, had minimai swelling, over the medial aspect of the ankle with no local tenderness. You advised rest, ice application and elevation. Which of the below will help the pain? A. Paracetamol B. Non steroidal anti inflammatory drug, ©. Paracetamol and an opioid D. Functional physiotherapy Une femme enceinte agée de 24 ans dans son dernier trimestre de grossesse subit une entorse de la cheville. Elle rapporte une douleur severe de grade 8/10. elle était capable de marcher 10 pas, elle a un gonflement minime au niveau de la partie interne de la cheville sans douleur localisée. Vous lui conseillez un repos au lit, une application de glace ot surélévation du membre. Quel est parmi les suivants celui qui va aider a calmer la douleur? A. Paracetamol B._Anti-inflammatoire non stéroidien C. Paracetamol et opiace D._Physiothérapie fonctionnelle Gab SLE Ope a OBS AALS oI ab Ge Gols Jatt Ge I OP EUG V/A ca ll Ga Ss Ua BH AS ayy Lely Habs V6 gl PL O98 Oe Jats go che aa ED ety Hat Jb Lena «peti Haar pad wees J Ly lll Ge gl lining Soke LA plate oe gh ae BO | 98 Bas Stal $C baie D 153 Which of the following medications has been shown to reduce mortality rates in patients with type IK diabetes? Exenatide Rosiglitazone (Avandia) SitagispinGanwvia) Metformin(Glucophage) Sulfonylurea mone Quelle est parmi tes medicaments suivants celui qu’on a demontre qu'il reduit le taux de mortalite chez ies patients ayant un diabete de type IT? A. Exenatide B. Rosiglitazone (Avandia) C. Sitaglipine(anuvia) D. Metformine(Glucophage) ¥, Sulfonyluree B-65 an OE Mo gl bs Ge Sith aes Se SLID Jane TT de pS asa, OIE 5378 Bye aC sp sheD boy SiplleE

You might also like